MCQ - All Topics

Lakukan tugas rumah & ujian kamu dengan baik sekarang menggunakan Quizwiz!

Defendant was arrested for the robbery of a liquor store. As the police were transporting Defendant to the police station to be booked, they read him his rights. Defendant then stated, "I know my rights by heart by now. Call my lawyer cause I ain't talkin' without him."The police finished the booking procedure. As they are walking Defendant out to transport him to the county jail, a witness from the liquor store that just finished giving a statement walked by. Defendant then yelled, "I know my rights! You can't just walk me by her for an ID without my lawyer being here. I asked for him; you have to do a fair line up!"At Defendant's trial the prosecution moved to admit Defendant's statement made at the police station. The statement is: A) Admissible, because Defendant volunteered the statement. B) Admissible, because it did not occur at a critical stage in the proceedings, so the request for a lawyer was not binding. C) Not admissible, because Defendant requested a lawyer be present. D) Not admissible, because the identification was illegal.

A) Admissible, because Defendant volunteered the statement.

Patience owned and operated Peaceful Pastures, a small but exclusive nursing home. Patience decided to develop a rose garden on the grounds and planned a walkway through the gardens where the residents could stroll in the cool evening hours. Patience and Duffy, the owner of Duffy's Concrete Supply, entered into a written contract whereby Duffy agreed to provide one truck load of "sunset rose" colored concrete to Patience for construction of the walkway which was to measure 3 feet wide, 30 feet long and 3 feet deep. Patience agreed to pay $45 per yard of concrete. When Duffy fails to perform, Patience sues him for breach of contract. Duffy's defense is that there is no way to measure Patience's damages because the number of yards of concrete was not specified. Duffy points out that he has trucks which carry five yards as well as trucks which carry ten yards.If Patience offers to show that in the building trade it is accepted that a cubic yard equals 3' x 3' x 3', and that the planned pathway would require 10 cubic yards of concrete, the court should find the evidence is: A) Admissible, because trade usage or custom is relevant to supply a missing contract term. B) Admissible, because ther

A) Admissible, because trade usage or custom is relevant to supply a missing contract term.

Police set up a surveillance of Shirley's home. They suspected that Shirley was a major distributor in the local heroin trade. One evening, the police observed Shirley load a couple of suitcases into Delbert's trunk. Shirley then leaned into the passenger window and whispered something to Delbert.The police followed Delbert for 10 blocks and then executed a traffic stop. When they called in Delbert's driver's license number, they were informed that Delbert had an arrest warrant for unpaid child support. The police asked Delbert to exit the car and placed him in handcuffs. They then searched the car and found a small bag of marijuana in the glove compartment. They found nothing else in the passenger compartment of the car.At trial, Delbert's lawyer moved to suppress the marijuana seized from the glove compartment. The judge should rule that the marijuana should be: A) Admitted, because it was incident to the arrest for unpaid child support. B) Admitted, because they had probable cause that contraband would be found in the car. C) Suppressed, because contraband is not usually connected to child support violations. D) Suppressed, because Delbert was not in the car at the time of the search.

A) Admitted, because it was incident to the arrest for unpaid child support.

Under which of the following circumstances is the defense of necessity most likely to succeed? A) Defendant and Victim were painters for the Golden Gate Bridge. While Victim was moving past Defendant from one spot high on the north tower to another, having negligently unhooked his safety line, a powerful earthquake struck. Victim was just able to hook his safety line to Defendant's safety harness before the violent shaking of the bridge threw them both off into the air. Defendant, hanging by his safety line, felt the added weight of Victim begin to tear his safety harness, which, if completely ruptured, would send both into the waters of the Golden Gate far below. In order to prevent his harness from completely separating, Defendant unhooked Victim's safety line, sending Victim to his death. B) Defendant's husband, Victim, suffering from an excruciating form of terminal cancer which robbed him of control over his body and caused continual, untreatable pain, begged her to end his life. Defendant turned off all of the medical support equipment which maintained Victim's life, and after a few minutes he died. C) Defendant, Victim, and Other managed to reach an inflatable lifeboat after the

A) Defendant and Victim were painters for the Golden Gate Bridge. While Victim was moving past Defendant from one spot high on the north tower to another, having negligently unhooked his safety line, a powerful earthquake struck. Victim was just able to hook his safety line to Defendant's safety harness before the violent shaking of the bridge threw them both off into the air. Defendant, hanging by his safety line, felt the added weight of Victim begin to tear his safety harness, which, if completely ruptured, would send both into the waters of the Golden Gate far below. In order to prevent his harness from completely separating, Defendant unhooked Victim's safety line, sending Victim to his death.

A manufacturer invited bids from several coal companies to supply the manufacturer with 10 tons of metallurgical coal. A supplier offered to do so for $750,000. This was an excellent price because it was substantially under the market price for metallurgical coal. The manufacturer immediately entered into a written agreement to purchase the coal from the supplier, with delivery to be on or before August 10. Several weeks before the coal was to be delivered, the supplier notified the manufacturer that it had made an arithmetical error in pricing the coal and that the price should have been $1,000,000. The supplier requested a price increase to $825,000, which the company confirmed in writing. When the manufacturer received the supplier's bid for $750,000, the manufacturer had no reason to believe that the supplier had made an error in its calculations. The modification by which the manufacturer agreed to pay $825,000 is: A) Enforceable, because the request for the modification was made in good faith. B) Enforceable, only if the supplier can establish that enforcement of the contract at the $750,000 price would be unconscionable. C) Unenforceable, because the supplier was under a pre-exist

A) Enforceable, because the request for the modification was made in good faith.

Felix has just been arrested for a robbery. Knowing that he was a "third strike" candidate for an enhanced sentence, Felix told the police that he had always sold the stolen merchandise from his robberies to Donna, a suspected fence, and that he would help them arrest Donna for receiving stolen property.The police agreed to work with Felix. They drove him to Donna's house and sent him in with merchandise that was electronically marked and had transmitters secretly attached. Felix was under police surveillance from the time he left the police van to the time he went in the door of Donna's house, and from the time he left the door of Donna's to the time he entered the police van. When he returned to the van Felix no longer had the merchandise in his possession.The police immediately moved in with the intention of arresting Donna for receiving stolen property. They knocked and announced their presence, but received no response. The police then knocked down the door and entered Donna's house, finding the marked merchandise and numerous other items of expected stolen property. However, Donna was not present. She had exited the house by the back door to go get dinner. Donna was subsequently ar

A) Grant the motion, because there were no exigent circumstances to justify the entry into Donna's home.

A nonresident of New York filed a shareholder's derivative suit in a New York court, naming as defendants six present or former corporate officers, all nonresidents of New York. The lawsuit claims that the individual defendants had violated their duties to the corporation by causing it to engage in activities that resulted in corporate liability and substantial fines. None of these actions occurred in New York. The plaintiff also filed a motion for sequestration of New York property of the individual defendants. The property consisted solely of stock in the corporation. None of the certificates representing the stock were physically present in New York. However, because the corporation itself was incorporated in New York, a state law made New York the location of stock ownership. The defendants entered a special appearance, arguing that the New York court did not have jurisdiction over the suit.Which of the following is the most accurate statement regarding the New York court's jurisdiction? A) It likely does not have jurisdiction because there are not sufficient contacts between the defendants and the state. B) It has jurisdiction because the plaintiff sequestered New York property. C)

A) It likely does not have jurisdiction because there are not sufficient contacts between the defendants and the state. In actions involving attachment of real or personal property as part of the relief requested, the federal court located in the state in which the property is situated has quasi in rem jurisdiction over the real and personal property being attached. However, minimum contacts are still required in order for the court to exercise quasi in rem jurisdiction. When the dispute concerns the parties' rights in the property itself, the presence of the property in the state provides sufficient minimum contacts. However, when the dispute is unrelated to ownership of the property, minimum contacts must be shown between the defendant and the forum state. Here, the suit was not over the ownership of the stock itself, but rather, was over the possible misfeasance of the individual defendants. Consequently, the plaintiff must still show minimum contacts between the defendants and the state and the facts do not provide any contacts other than the stock ownership. As a result, the court likely does not have jurisdiction.

A plaintiff filed a negligence suit against a grocery store in federal court because she was injured when she slipped on some grapes that were left on the floor of one of the aisles. After trial, the jury found in the plaintiff's favor. The defendant grocery store believed that the plaintiff's evidence against it was too weak to support the verdict, so the defendant moved for judgment as a matter of law 28 days after judgment was entered. In their motion, the defendant argued that the plaintiff did not provide sufficient evidence that the grape was the cause of the plaintiff's fall.Should the court grant the motion? A) No, because the defendant did not previously request judgment as a matter of law. B) No, because the motion had to be filed within 20 days after the judgment was entered. C) Yes, because it was filed within 28 days after the judgment was entered. D) Yes, because the defendant was claiming the plaintiff failed to meet her burden of proof.

A) No, because the defendant did not previously request judgment as a matter of law.

Stan's restaurant had ceased to be profitable when Vivian opened her Country Kitchen Restaurant across the street. Stan projected that he would go bankrupt in three months unless his patronage immediately rose to the levels he had enjoyed prior to the opening of Vivian's restaurant. Stan asked his bookie if he (the bookie) knew anyone who would do a job, no questions asked, for cash, and the bookie said to contact Pierce at a certain bar. Stan went to the bar, met Pierce, and after buying drinks offered Pierce $5,000 to burn down Vivian's restaurant. Pierce, an undercover police officer, immediately arrested Stan. Stan was subsequently charged with solicitation to commit arson. The jurisdiction has not altered the common law by any statute. A properly instructed jury should find Stan: A) Not guilty, because Pierce could not commit arson by performing the requested act. B) Not guilty, because there can be no solicitation when the person solicited is a police officer. C) Guilty, because solicitation is complete whether or not the person solicited actually carries out the crime. D) Guilty, because he offered to pay Pierce to commit arson.

A) Not guilty, because Pierce could not commit arson by performing the requested act.

A man was a casual acquaintance whom a woman had met at the Marina. The man would infrequently ask the woman to accompany him on a cruise around the bay in a cabin cruiser. On one such trip, the woman remarked that since the man used the boat so seldom, he should sell it, and gave the man the name and phone number of a prospective buyer. The man sold the boat to the buyer for cash, and then disappeared. Unknown to the woman, the man had merely been using the cabin cruiser with the true owner's permission. If the woman is charged with aiding and abetting the man in committing the crime of embezzlement of the cabin cruiser, which of the following verdicts should the jury properly return? A) Not guilty, because she believed that the man was the owner of the cabin cruiser. B) Not guilty, because her acts were not a legal cause of the crime. C) Not guilty, because she was not present when the crime occurred and thus cannot be a principal in the second degree. D) Guilty, because she counseled and assisted the man, and her mistake of fact regarding the true ownership of the boat is no defense.

A) Not guilty, because she believed that the man was the owner of the cabin cruiser.

A corporation owned two office buildings in Metropolis that were located directly across a city street from each other. Pursuant to city building codes, the corporation had constructed an enclosed metal walkway spanning the street that permitted workers to walk from one building to another at the second-floor level. The corporation had negligently failed to maintain the metal support structure of the walkway so that its load-carrying capacity was reduced by half.One day, an applicant was interviewed for a job by an executive of the corporation. The executive invited the applicant to have lunch at the company cafeteria across the street, and she led the applicant to the metal walkway between the two buildings. At the opposite walkway entrance, an office supplies vendor was pushing an exceptionally large cart full of laser printer paper across the walkway. The weight of the heavy paper and the weakened condition of the supports caused the walkway to collapse just as the cart and the applicant reached the center of the walkway. The applicant was thrown to the street below and injured severely. The collapse would not have occurred if the walkway had been properly maintained, nor would the we

A) Prevail against the vendor and the corporation, each being jointly and severally liable for 100 percent of his damages.

Able and Baker were leaving the gym where they played handball when a young woman, Clara, confronted them. She held a large clutch purse in her left hand; her right hand was inside the purse. "I have a pistol in my purse," she said to Able. She pointed the purse at Baker and said to Able, "Give me your watch, rings and wallet or I'll kill your friend." Able immediately complied, dropping all of his jewelry plus his wallet into her purse, which she held so that he could not see inside. As the woman turned to leave, a police officer on walking patrol rounded the corner. Able and Baker both called for help, and the officer arrested Clara. A search of her purse for weapons revealed that she had had no gun or any other weapon.If Clara is prosecuted for robbery, what should the result be? A) She should be convicted of robbing Able, because she took Able's property from him by threat of force. B) She should be convicted of robbing Baker, because the taking of Able's property is considered as taking from Baker's "person or presence." C) She should be found not guilty, because she did not threaten Able, whose property she took, and did not take property from Baker, whom she threatened. D) She sho

A) She should be convicted of robbing Able, because she took Able's property from him by threat of force.

A landowner and a neighbor owned adjacent tracts of land. The landowner told the neighbor that the neighbor could share the use of a private road on the landowner's land so that the neighbor could have more convenient access to the neighbor's land. Maintenance on the road was done by the landowner, not by the neighbor, even though the neighbor used the road daily. After three years, the landowner sold the land to a buyer. The buyer immediately notified the neighbor to stop using the road.When the neighbor sues the buyer to determine the neighbor's right to continue using the road, who is likely to prevail? A) The buyer, because the buyer could terminate the grantee's license at any time. B) The buyer, because oral licenses are not valid. C) The neighbor, because the neighbor's daily use of the road was open and notorious, meaning that the buyer should have reasonably known of the neighbor's use when the buyer purchased the land. D) The neighbor, because the buyer is estopped from terminating the use of the road by the neighbor.

A) The buyer, because the buyer could terminate the grantee's license at any time.

A contractor agreed to build a restaurant for a developer in accordance with detailed plans and specifications. In return, the developer agreed to pay the contractor $100,000 for the restaurant upon completion. The written contract included the following provision: "Payment to the developer is expressly conditioned on the use of South African rose marble to floor the foyer." While the restaurant was under construction, the President of the United States, acting under proper legal authorization, banned all further trade with South Africa. Since no South African rose marble was otherwise available, the contractor used a similar fine quality Italian marble in the foyer instead. Other than the floor, the restaurant was completed according to the plans and specifications contained in the contractor-developer contract.If the developer refuses to pay the contractor because of his failure to meet the requirement that South African rose marble be used in the foyer, and the contractor sues on the contract, which of the following is the contractor's strongest argument? A) The requirement is an express condition that is excused by the supervening illegality resulting from the President's ban on impo

A) The requirement is an express condition that is excused by the supervening illegality resulting from the President's ban on importation

A starlet's claim to fame was her wild red hair which she claimed was natural. A photographer followed the starlet to her hairdresser's house and, using a drone equipped with a camera, photographed through the second story window the starlet having her hair colored. If the starlet sues the photographer on the basis of invasion of privacy, what is the likely result? A) The starlet should prevail. B) The starlet should not prevail because the photograph accurately portrays what the camera saw. C) The starlet should not prevail because she was not in her own house. D) The starlet should not prevail because she is a public figure.

A) The starlet should prevail.

A plaintiff sued a defendant, alleging that the defendant violated the plaintiff's trademark rights by selling blankets under a company name that is similar to plaintiff's company name. The plaintiff served interrogatories on the defendant, asking him to explain the organization of his company, its profits and losses, and its sales techniques.Which statement regarding the interrogatories is correct? A) They must be answered within 30 days of service. B) The plaintiff is limited to 30 questions. C) They need not be answered under oath. D) They need not be answered in writing.

A) They must be answered within 30 days of service.

A woman owned a pet llama which she housed in a barn and fenced corral. The woman chose a style of fencing which was inadequate to contain the llama. Within days after acquiring it, the llama easily scaled the fence and strayed onto an owner's property. The next day, when the woman discovered the llama was missing, she tracked it onto the owner's property. In the process of recapturing the llama, the woman knocked over several fence posts of a corral where the owner kept various barnyard animals. If the owner sues the woman for trespass, will he prevail? A) Yes, because the woman was not privileged to enter onto the owner's property. B) Yes, because the woman damaged the fence posts. C) No, because the woman was privileged to enter the owner's property at a reasonable time and place to reclaim her personal property. D) No, because the woman was privileged to enter by private necessity.

A) Yes, because the woman was not privileged to enter onto the owner's property.

A woman learned that her sister had obtained a new wide-screen television on their father's account at the local appliance store. The woman believed that the father had not authorized this purchase, and she decided to spare her father any pain in discovering what her sister had done. Therefore, the woman told the appliance store owner: "If you will not seek payment from my father, I will pay for the television." A few months later, the father passed away.If the appliance store owner did not attempt to collect the purchase price for the television from the father prior to his death, would the appliance store owner succeed in an action against the woman for the purchase price? A) Yes, because the woman's promise was supported by a bargained-for exchange. B) Yes, because the appliance store owner detrimentally relied on the woman's promise. C) No, because the woman at most had only a moral obligation to pay the debt. D) No, because the appliance store owner's claim against the father terminated on his death.

A) Yes, because the woman's promise was supported by a bargained-for exchange.

Although Tyler was a skilled tailor whose work was always in demand, he lacked any ability to manage his money. Tyler offered to make Wallace a three-piece wool suit in a workmanlike fashion by October 1, if Wallace on that day would pay a $500 debt that Tyler owed to Elsie. When Wallace agreed, Tyler informed Elsie of the agreement. After Tyler finished the suit, Elsie demanded payment, but Wallace refused to pay. Assume the suit was poorly made. Elsie sues Wallace for $500. Does the poor quality of Tyler's work provide Wallace with a defense to Elsie's lawsuit? A) Yes, without qualification. B) Yes, but only if Wallace can show that Elsie knew Tyler's work was of poor quality. C) No, without qualification. D) No, provided Elsie changed her position in reliance on the Tyler-Wallace contract.

A) Yes, without qualification.

An issue arose during a jury trial as to whether a witness should be qualified as an expert for the plaintiff. Both sides had an opportunity to examine the witness, and the judge ultimately denied the witness as an expert, but told the attorneys that it was a close call. After the jury returned a verdict for the defendant, the plaintiff contemplated filing an appeal based in part on the judge's disqualification of the plaintiff's witness as an expert.Which of the following is the standard of review that would be used to decide the issue on appeal? A) Against the great weight of the evidence. B) Abuse of discretion. C) De novo. D) Clearly erroneous.

B) Abuse of discretion.

Cruiser and Flash were each driving down Main Street in their distinctively painted, carefully-restored old American automobiles when Cruiser's vehicle bumped lightly into Flash's car, scraping away some paint. Cruiser and Flash both got out of their cars and examined the damage, after which Flash made disparaging remarks about Cruiser's car. Cruiser slapped Flash in the face; Flash punched Cruiser in the nose. Cruiser seized Flash in a headlock, and started to ram Flash's head into the side of Flash's car. At that moment, Flash's brother, Glitter, who had just driven by, leaped from his car, pulled a knife, and held it to Cruiser's throat, saying, "Let him go or I'll cut you." Cruiser released Flash, but subsequently sued Glitter for assault. What should be the result of the suit? A) Cruiser wins, because Glitter threatened him with deadly force. B) Cruiser wins, unless ramming Flash's head into a car is considered a deadly attack. C) Glitter wins, because Cruiser started the fight by slapping Flash. D) Glitter wins, because he and Flash are relatives.

B) Cruiser wins, unless ramming Flash's head into a car is considered a deadly attack.

The police received a call that there had just been an armed robbery of a local grocery store. The first officer to respond got a description of the robber and broadcast it out over the radio.George, a patrolling officer, saw Defendant walking fast and looking back and forth. His clothing, build, and features fit the basic description given by the store clerk. George stopped Defendant and yelled, "Put your hands on your head and get on the ground." Defendant complied and stated, "What's this about? I'm on my way home from work. This is how you treat a hard-working citizen?" George then frisked Defendant and found items he believed had been taken in the robbery. George arrested Defendant, put him in his police car, and drove him to the grocery store. At the grocery , the clerk was brought out to look at him. She immediately started crying and stated, "Oh my god, that's him!" Defendant was then taken to the police station and held overnight. The next morning, Defendant appeared before Judge without a lawyer, and Judge refused to set bail, stating, "You are a danger and a menace to society. I am going to make sure that while you are awaiting trial you cannot hurt anyone else."With reference

B) Defendant's right to bail was violated.

Merton was manager of a fast food restaurant. One of his duties was to gather all money received during the preceding 24-hour period at 3:00 p.m. each weekday and deposit that money in the restaurant account at the local bank. Merton was in financial difficulties but had practiced a method of playing casino blackjack that he felt would enable him to win consistently; he only needed a stake to apply his method. One Friday, instead of depositing the cash receipts for the day at the bank, Merton took the restaurant cash and drove across the state line to a city where gambling was legal. Merton played blackjack using his system for the entire weekend, pausing only to sleep, but on Sunday evening he was only a few hundred dollars ahead of the amount with which he had started. Merton drove back to his home and the next morning deposited in the restaurant bank account an amount equal to the Friday restaurant receipts, plus an appropriate amount of interest.Based on these facts, Merton could be convicted of: A) No crime. B) Embezzlement. C) Attempted larceny. D) Larceny.

B) Embezzlement.

Fourteen years ago, two friends moved onto unused hillside pastureland owned by a landowner they didn't know and built a small cabin on it. Nine years ago, the friends planted grape vines, built other buildings, and began operating a vineyard. The two continued their grape growing and winemaking until earlier this year when one of the friends died intestate, leaving a daughter as his heir.The landowner recently discovered the grape vines and vineyard buildings on the land. The adverse possession period in this jurisdiction is 10 years.If the landowner brings an action to quiet title, for whom should the court rule? A) For the friend who is still alive, as the surviving adverse possessor. B) For the daughter of the deceased friend and the friend who is still alive, because the two friends acquired title as tenants in common. C) For the landowner, because title cannot be claimed by two adverse possessors at the same time. D) For the landowner, because the friends' use of the land changed.

B) For the daughter of the deceased friend and the friend who is still alive, because the two friends acquired title as tenants in common.

A businessman owned a restaurant which catered to families with children. The restaurant featured a special enclosed area where the children could sit separate from their parents, ordering from a special children's menu. Outside the children's dining area was a play yard containing various play structures and objects, which the children could use after finishing their meal, while their parents ate at leisure.The businessman was aware that several children who lived nearby were in the habit of climbing over the low fence which surrounded the play yard, in contravention of the signs warning that use was limited to restaurant customers, and playing on the facilities. The businessman had decided that, so long as the neighborhood children were not disruptive and did not displace customers, it was better for neighborhood relations and thus for business if he did not eject them.Unknown to the businessman, the large metal spring which held up a hobby horse in the play yard had prematurely rusted almost completely through, and lost most of its structural integrity. The rust was not visible through the paint on the hobby horse. An eight-year-old boy from the neighborhood, who had climbed into the

B) Judgment for the businessman, because he was not actually aware of the rusted hobby horse support, and the boy was a trespasser.

Melvin purchased a heavy-duty water heater manufactured by Heatco to use as a backup to his solar water heater. The solar heater only worked on sunny days, so Melvin wanted to link the water heater and the solar heater so that the water heater tank would hold solar-heated water when available but activate to provide gas-heated water on overcast days.Melvin asked his neighbor Paul to assist him with the installation of the water heater. When the installation was complete, Melvin and Paul got a couple of beers and a couple of chairs and sat down in front of the heater to see how their efforts succeeded. Melvin opened the valve, which permitted water to enter the tank from the solar unit. Because of a defective pressure relief valve in the water heater, the valve failed to activate until the water from the solar unit entering the water heater became superheated. The link then suddenly exploded, seriously scalding Melvin and Paul.Paul brought an action against Heatco for the physical injuries he suffered as a result of the water heater explosion. Which party will probably win this lawsuit?In an action by Melvin against Heatco, who will prevail? A) Melvin, because misuse is never a defense to

B) Melvin, because he was injured by the defective water heater.

Margaret owns and operates an au pair service. She recruits individuals, generally young women, from France and England who would like to come to the United States and work in an American household taking care of children. Margaret arranges for the young women to come to the United States and finds them a position. In exchange for their work, the young women receive room and board and a small monthly stipend. Margaret arranged for Adele to come to the United States and work for Lillian for a six-month period. Shortly after Adele arrived and began working for Lillian, she informed Margaret and Lillian that she was unbearably homesick and quit to return home to France.If Margaret and Lillian decide to file suit, who can obtain an order of specific performance? A) Either Margaret or Lillian. B) Neither Margaret nor Lillian. C) Margaret only, because she arranged for Adele's position with Lillian. D) Lillian only, because she is the one for whom Adele agreed to perform services.

B) Neither Margaret nor Lillian.

A woman applied for a job as a county correctional officer. After she failed to get the job, she sued the county for sex discrimination. At trial, the court found that she was the only applicant who was asked questions about her future plans concerning raising children, and members of the all-male hiring committee had expressed reservations about hiring a woman for the position. The court ruled in favor of the woman. The county appealed, arguing that there were non-discriminatory reasons for hiring the man that had ultimately been hired for the job: he had had 8 years more experience, had already done the job for another larger jail, and had the highest scores on the physical fitness test that all applicants had to take. The appeals court reversed, finding that the county had several non-discriminatory reasons for hiring the man over the woman. The appeals court's decision made clear it simply disagreed with the lower court's interpretation of the facts in the case.Did the appeals court rule correctly? A) No, because the question of whether to hire someone is a question of fact. B) No, unless the trial court's findings of fact were clearly erroneous. C) Yes, because there was significant

B) No, unless the trial court's findings of fact were clearly erroneous.

Vic left for a two-day business trip, but never returned. Vic's fiancee reported him missing after five days. The police then found Vic's car abandoned in a parking lot about three hours from home. They also found his wallet, briefcase, and his shirt with bloodstains on it in a commercial trash bin in the parking lot. Believing they were investigating a homicide, the police started canvassing the area. They stopped Defendant, who they observed standing near the parking lot, watching all the activity by the police. In response to their questioning, Defendant said that he had met Vic three days prior and spent the night drinking in a bar with him. Defendant further stated that Vic told him that he was heading out west to start a new life and that it was his last night in town. Defendant then said the Vic immediately left and he never saw him again.Defendant was arrested and charged with Vic's murder. During the trial, Defendant took the stand and testified that he never met Vic, doesn't know who he was, and never had any contact with him under any circumstances. The prosecution then attempts to present Defendant's statements made to the police detailing his night with Vic. What is the like

B) Overruled, because the statements are being used for impeachment.

In a state court of general jurisdiction, a plaintiff sued a corporation for personal injuries she received in an auto collision with a truck driven by a corporation truck driver. The plaintiff personally served the corporation's president by handing the summons and complaint to them the day before the president retired. In the excitement of her retirement, the president neglected to deliver the papers to anyone else at the corporation. A default judgment was entered against the corporation. Other than service upon the president, the corporation never received notice of the pending lawsuit prior to the entry of a default judgment. The corporation now moves to quash service of process. Assume that any applicable state law pertaining to service of process is identical to the Federal Rules of Civil Procedure.Which of the following is true? A) Service should be quashed, because the president was not an officer of the corporation at the time an answer was due. B) Service should be quashed, because the summons and complaint were served by the plaintiff. C) Service should not be quashed, because the corporation received actual notice of the pending lawsuit. D) The corporation's motion should be

B) Service should be quashed, because the summons and complaint were served by the plaintiff.

Sweet was enraged when she learned that Muller had been carrying on an affair with her (Sweet's) husband. Sweet went to her father's house and took his shotgun from the gun safe in which it was stored. She then drove to Muller's house, where she knew Muller was participating in a regular Wednesday afternoon bridge game. Sweet intended to shoot and kill Muller, but as she drove onto the street where Muller's house was located she was stopped and arrested by the police, who had been alerted by her father (to whom she had confided her intentions).Sweet was prosecuted for attempting to violate a statute which made it a felony to "enter onto the property of another person for the purpose of committing murder, attempted murder, assault with a deadly weapon, mayhem, rape or kidnapping." What should be the outcome of this prosecution? A) Sweet should be found guilty, because the statute is a public safety measure which she violated by arming herself with the shotgun. B) Sweet should be found guilty, because she was stopped by the police just short of completing the charged offense. C) Sweet should be found not guilty, because she cannot be convicted simply of having a "guilty mind." D) Sweet sho

B) Sweet should be found guilty, because she was stopped by the police just short of completing the charged offense.

The plaintiff was injured at a carnival located in the Western District of Missouri. The carnival was owned by Corporation, which was incorporated in Delaware and had its headquarters in the Eastern District of Iowa. The plaintiff lived in the Eastern District of Connecticut, and wanted to sue Corporation in the Eastern District of Connecticut for $1250,000 in damages.Where would venue not be proper for this action? A) The Western District of Missouri. B) The Eastern District of Connecticut. C) The Eastern District of Iowa. D) The District of Delaware.

B) The Eastern District of Connecticut.

Crystal is an avid vegetarian who avoids contact with any animal substance. She is a known member of the radical group "Animals First!" and is a well-known speaker throughout the country. She has appeared in various publications to preach the praise of animals and the evil of using animals to further human enjoyment.The Times recently published a photograph that appeared to show Crystal inside a McDonald's restaurant eating a hamburger with the caption, "A well-known vegetarian takes a break." As a result, Crystal was ejected from the group of pet activists and shunned by her peers. In fact, the photograph did not depict Crystal, but another person who looked almost identical to Crystal. Crystal wants to sue The Times for defamation.Crystal will not prevail in the lawsuit unless she can prove that: A) The Times sold more issues as a result of the appearance of the photograph. B) The Times failed to research if the photograph was accurate. C) She suffered actual damages as a result of the publication of the photograph. D) She was not in a public space of the restaurant.

B) The Times failed to research if the photograph was accurate.

An owner is the builder-developer of a group of recreational homes in a resort area. The two hundred homes are of four basic styles and represent the last large development to be permitted in the area due to overburdened water and sewage systems. The homes feature high beamed ceilings, indoor spas, ski storage rooms, lake views, and fireplace inserts.A buyer decided to sell her home and purchased one of the owner's units. The buyer moved into her new home a week before Thanksgiving. Temperatures were already getting cool enough at night to warrant a fire in the fireplace. The fireplace insert seemed to work well. The buyer stoked up a fire she thought would last for several hours and warm the house, then went to bed. However, a defect in the chimney allowed exhaust smoke to seep into the upstairs bedroom where the buyer was sleeping, causing her moderate injury from smoke inhalation.If the buyer sues the owner for her injuries, who will prevail? A) The buyer, but only if the owner failed to use reasonable care in installing the fireplace insert. B) The buyer, because a defect in the residence caused her injury. C) The owner, if the county found the residence in compliance with code requi

B) The buyer, because a defect in the residence caused her injury.

The owner of Greenacre conveyed Greenacre by quitclaim deed to her son and daughter "as joint tenants with right of survivorship." Thereafter, the son executed a valid will, leaving his interest in Greenacre to the college he attended. Later the daughter executed a valid will, leaving her interest in Greenacre to her friend. The daughter died and then the son died. Neither the son nor the daughter married or had any descendants. The friend survived and the college is still in operation.After the probate of both wills, who owns Greenacre? A) The friend owns Greenacre in fee simple. B) The college owns Greenacre in fee simple. C) The friend and the college own Greenacre as joint tenants with right of survivorship. D) The friend and the college own Greenacre as tenants in common.

B) The college owns Greenacre in fee simple.

A contractor was hired by the city to build an elementary school. The contractor hired a subcontractor to pave the playground for the school. The contractor agreed to pay the subcontractor $100,000 for the job. The subcontractor spent $80,000 on the job and completed 60% of the project, for which he was paid $60,000 by the contractor. The subcontractor then repudiated the contract without justification. The contractor hired another company to complete the job at a cost of $60,000. The contractor sued the subcontractor for damages, and the subcontractor counter-sued for restitution.What may each party recover? A) Neither party may recover anything because, although the subcontractor materially breached the contract, he conferred a benefit of $20,000 on the contractor. B) The contractor may recover $20,000, but the subcontractor may not recover anything. C) The contractor may recover $60,000, the cost of hiring the company which finished the paving, but the subcontractor may not recover anything. D) The contractor may recover $20,000, and the subcontractor may recover $20,000, the value of the benefit conferred on the contractor.

B) The contractor may recover $20,000, but the subcontractor may not recover anything.

A plaintiff sued a defendant for injuries sustained when he fell while getting off a ride at the defendant's waterslide park. The device holding the raft had prematurely broken open, causing the plaintiff to go down the slide when the plaintiff was not completely in the raft. At the plaintiff's suggestion, his girlfriend immediately took pictures of the broken device. Because the park was still open, the defendant hired a contractor to replace the device that same afternoon. In doing the work, the contractor had to completely disassemble the broken device. The plaintiff's attorney hired an expert to prepare a report regarding the defective nature of the device. The defendant moved to discover the report prepared by the expert and the pictures taken by the plaintiff's girlfriend of the broken device. The plaintiff claims both the pictures and the report are covered by the work-product privilege.How will the reviewing court decide the defendant's discovery request in light of the plaintiff's claim of work-product privilege? A) The court will hold that Amusement Co. may view the pictures and obtain the report. B) The court will hold that the defendant may view the pictures, but may not obta

B) The court will hold that the defendant may view the pictures, but may not obtain the report unless the expert will testify at trial.

After an uncle was diagnosed with a terminal illness, he executed a valid deed conveying his home to his nephew, who was then serving overseas in the military. The uncle handed the deed to his nephew's mother (the uncle's sister-in-law) and said, "I want [the nephew] to have my home. Please take this deed for him." Shortly thereafter, the nephew's mother learned that the uncle's death was imminent. One day before the uncle's death, the nephew's mother recorded the deed. The nephew returned home shortly after the uncle's death, learned about the deed, and took possession of the home. The uncle died without a will, leaving a son as his only heir. When the son claimed ownership of the home, the nephew sued to quiet title. Under the laws of the jurisdiction: (1) a deed may be recorded if it is signed by two witnesses; and (2) only two witnesses are required for a will to be properly executed.What would be the most likely reason for the court to rule for the nephew? A) The uncle's death completed a valid gift causa mortis to the nephew. B) The deed was delivered when the uncle handed it to the nephew's mother. C) The uncle's properly executed deed was effective as a testamentary document. D)

B) The deed was delivered when the uncle handed it to the nephew's mother.

A landowner owned a vacant tract of land. The southern boundary of the tract was a public highway. The tract was bounded on the west, north, and east by other tracts of land. The landowner conveyed the southern half of the land to a buyer by warranty deed. Soon thereafter, the landowner conveyed the northern half of the land to an investor by warranty deed.Some years later, the buyer died, leaving the still-vacant southern tract to his daughter. A few years after the buyer's death, as the adjacent tracts of land began to be platted for subdivisions, the investor sold the still-vacant northern tract to a developer.In preparation for subdividing the northern tract, the developer contacted the daughter to purchase the right to cross the southern tract. The daughter refused to sell the right to cross to the developer at any price.If the developer sues the daughter to establish the right to cross her tract, who will prevail? A) The developer, because the daughter unreasonably refused to sell the right to cross to the developer. B) The developer, because the requirements for an easement by necessity have been met. C) The daughter, because the developer was aware that the northern tract was lan

B) The developer, because the requirements for an easement by necessity have been met.

An aunt owned a vacant lot. She often told her friends and relatives that she planned to leave the lot to her nephew in her will. The nephew was aware of the aunt's statements. Prior to the aunt's death, the nephew conveyed the land to a friend for $15,000 by warranty deed. The friend did not conduct a title search before accepting the deed from the nephew. The friend recorded the deed the next day. Later that year, the aunt died. When the aunt's will was probated, it left the lot to the nephew. Both the nephew and the friend now claim ownership of the lot. The nephew has offered to return the $15,000 to the friend.Who now owns the vacant lot? A) The friend, because the friend recorded the deed prior to the aunt's death. B) The friend, because of the doctrine of estoppel by deed. C) The nephew, because the aunt was the owner of record at the time the nephew gave the deed to the friend. D) The nephew, because the friend assumed the risk by failing to conduct a title search.

B) The friend, because of the doctrine of estoppel by deed.

The owner of Blackacre mailed her friend a deed that conveyed Blackacre to the friend. The owner included a note to her friend that read, "I know you love hiking on Blackacre as much as I do, and I want you to have it. But please don't record this until after I die. If my son finds out, he will make my life miserable." The following day, the owner called her friend, told her friend that she was going to leave Blackacre to her friend in her will, and asked her friend to return the deed. Her friend said that she would destroy the deed and the owner said that that would work. A few days later, when the two spoke again, the friend falsely told the owner that she had destroyed the deed. A few weeks later, the owner died without a will, survived by her son as her sole heir. The day after the owner's death, the friend recorded the deed to Blackacre. Upon learning of the recording of the deed, the son brought an action to quiet title against the friend.Who will prevail? A) The friend, because the deed to Blackacre was recorded by her. B) The friend, because the deed to Blackacre was delivered to her. C) The son, because of the friend's bad faith in falsely reporting the destruction of the deed t

B) The friend, because the deed to Blackacre was delivered to her.

Three years ago, a grantee received a deed to land that included this language of conveyance: "To the grantee, the grantee's heirs and assigns; provided, however, that the grantee may not convey or otherwise transfer (except upon the death of the grantee) any interest in the land described herein for a period of 15 years from the date given on this deed."Earlier this year, the grantee entered into a contract to sell the land to a developer. The developer's title search revealed the quoted language in the grantee's deed. Based on this language, the developer refused to close the transaction. The contract is silent regarding the grantee's title obligations.If the grantee sues the developer for specific performance, who is likely to prevail? A) The grantee, because the contract did not obligate the grantee to provide marketable title. B) The grantee, because the deed's restraint on alienation is void as a matter of law. C) The developer, because the grantee's heirs were not parties to the contract. D) The developer, because the grantee cannot sell the land during the 15-year period.

B) The grantee, because the deed's restraint on alienation is void as a matter of law.

A developer bought land that was zoned for commercial uses, giving a mortgage secured by the land to an investor to fund the purchase. The mortgage contained this language: "If the mortgagor shall convey or otherwise transfer the mortgagor's interest without first obtaining the consent of the mortgagee in writing, then the entire principal balance remaining then unpaid shall be immediately due and payable at the option of the mortgagee." After making payments to the investor for several years, but without consulting the investor, the developer sold the land to an adjacent landowner. The developer and the landowner expressly agreed that the landowner would not assume and agree to pay the developer's debt to the investor. The deed given by the developer to the landowner stated that it was "subject to a mortgage to investor, recorded on [date], in the office of the county recorder of deeds." The landowner placed "no trespassing" signs on the land and made several payments to the investor, which the investor accepted. After neither the developer nor the landowner made the last three payments, the investor sued the landowner for the amount of the missed payments.Who will prevail in the lawsui

B) The landowner, because the landowner did not assume and agree to pay the developer's debt to the investor under the mortgage.

A landowner owned land with a public highway as its southern boundary. A neighbor owned the land immediately to the north of the landowner's land. The landowner's predecessor in title had given to the neighbor's predecessor in title an easement allowing the landowner's land to be crossed in order to access the public highway. The neighbor's land also has public road access along its east side.Earlier this month, the landowner installed a series of solar panels on the land (even though the landowner remains an electrical customer). The placement of the panels on the part of the land that was subject to the easement was critical to achieving the maximum generation of electricity by the panels. However, that placement of the panels also blocks the neighbor's ability to use the easement.A local ordinance authorizes the use of solar panels. There are no other applicable laws. If the neighbor sues to have the solar panels removed, who will prevail? A) The neighbor, because the landowner has access to other electrical service. B) The neighbor, because the easement remains valid. C) The landowner, because local ordinance authorizes the use of solar panels. D) The landowner, because the neighbor

B) The neighbor, because the easement remains valid.

An owner of land sold that land to an investor with a written contract that made no mention of the quality of title to be conveyed. At the closing, the owner delivered a warranty deed to the investor. Over the next year, the land's value rose dramatically. The investor sold the land to a developer with a written contract that expressly provided that the investor would convey a marketable title to the developer. Prior to closing, the developer's attorney discovered that the title to the land (1) was not now marketable and (2) had not been marketable when the original owner had sold it to the investor. Based on this information, the developer refused to complete the sale. The investor sued the original owner for breach of contract, claiming damages from the owner's failure to convey marketable title, resulting in the loss of the sale of the land to the developer.Who will prevail and why? A) The owner, because she did not expressly agree to convey marketable title. B) The owner, because under the doctrine of merger her obligations under the contract related to title merged into the deed. C) The investor, because the law implies a covenant that the title will be marketable into such contract

B) The owner, because under the doctrine of merger her obligations under the contract related to title merged into the deed.

Fifteen years ago, a neighbor built a fence along what the neighbor believed to be the property line that the neighbor's land shared with that of an adjacent landowner. Eight years ago, the neighbor sold her land to a purchaser who continued to maintain the fence. Last year, the landowner sold his land to a buyer. When the buyer had the land surveyed in preparation for building a new garage, the survey revealed that the fence built by the neighbor was actually two feet across the property line on the buyer's property.The adverse possession period in the jurisdiction is 10 years. If the buyer brings an action against the purchaser to quiet title, who will prevail? A) The purchaser, because the landowner reasonably should have known about the encroachment. B) The purchaser, because the requirements for adverse possession have been met. C) The buyer, because the neighbor was mistaken about the location of the property line and did not intend to encroach on the landowner's land. D) The buyer, because the neighbor's sale of her land to the purchaser restarted the adverse possession period.

B) The purchaser, because the requirements for adverse possession have been met.

In her prosecution for murder of Police Officer Ben, Diane claimed that she had acted in an extreme heat of passion because Officer Ben had beaten her with a nightstick while making an otherwise lawful arrest on an outstanding arrest warrant for numerous parking violations. Diane introduced evidence supporting her claim and argued that she should be convicted of voluntary manslaughter rather than murder. The jurisdiction statutorily codified the common law definition of the crime of murder as the homicide of another human being, with malice, which is not justified, excused or mitigated. How should the jury be instructed as to the burden of proof regarding Diane's claim that her killing of Ben was mitigated by heat of passion? A) They should be told that the prosecution must prove by clear and convincing evidence that Diane was not acting in the heat of passion when she killed Ben. B) They should be told that the prosecution must prove beyond a reasonable doubt that Diane was not acting in the heat of passion when she killed Ben. C) They should be told that Diane must prove by clear and convincing evidence that she was not acting in the heat of passion when she killed Ben. D) They should

B) They should be told that the prosecution must prove beyond a reasonable doubt that Diane was not acting in the heat of passion when she killed Ben.

Duncan was attending the local county fair when he came upon an outdoor booth demonstrating the psychic abilities of "Yuri the Psychic." Duncan decided to stay for the demonstration, and stood at a spot just to the left of the stage. Yuri was purporting to bend silverware using only his psychic power, and became visibly annoyed when Duncan announced in a loud stage whisper that he had seen The Amazing Randi do the same spoon-bending as a sleight-of-hand trick on the Tonight Show several years before. Yuri's anger grew as Duncan similarly deflated his Incredible Floating Ball and Surgery Without Instruments demonstrations. Incensed beyond control, Yuri rose and extended his arms towards Duncan and shouted: "I shall use my psychic powers to burst the blood vessels of your brain!" Duncan drew a pistol from his waistband and shot Yuri in the head, killing him.At Duncan's trial for the criminal homicide of Yuri, Duncan testified that he shot Yuri because he believed it was necessary to prevent Yuri from killing him by psychically bursting the blood vessels in his brain. Duncan should be convicted of: A) Murder, because no one could honestly believe that Yuri had the psychic power to kill by b

B) Voluntary manslaughter, if the jury believes Duncan's testimony.

Garcia, a computer programmer and hacker, developed a program by which he could use the mainframe computer owned by his employer to raid the accounts of financial institutions which utilized electronic transfer of funds. Using the machine at work would leave traces by which he could be discovered, and his home computer was not powerful enough to run the program, so Garcia approached his friend Lou with a proposition: if Lou would give Garcia $20,000 to purchase a new computer which could run the account-raiding program, Garcia would give Lou $50,000 from the money he stole using the program. Lou gave Garcia $20,000 but Garcia had a change of heart; he never bought the new computer and never used his program to raid any bank accounts. Assuming that in the jurisdiction it is a crime to steal money by electronic means, at what point in the above facts could Lou be properly convicted of conspiracy to commit such electronic theft? A) When Garcia proposed the criminal scheme to him. B) When he gave the $20,000 to Garcia. C) Never, because he never agreed to participate in the electronic thefts. D) Never, because no electronic theft ever occurred.

B) When he gave the $20,000 to Garcia.

Phillip is the owner of a small winery. For several years he has been dumping grape sludge, which is the waste product of the fermenting process, in a pit on his land about 100 yards from Jennifer's property. The sludge continues to ferment, giving off a highly pungent and offensive odor. Jennifer finds it impossible to enjoy any outdoor activities in her yard because of the odor. If Jennifer's four-year-old daughter ran onto Phillip's land chasing a butterfly, fell into the sludge pit, and was injured, would she recover? A) Yes, but only if Phillip knew of her presence. B) Yes, if Phillip knew young children had entered his land before and were likely to be injured. C) No, because she came on the land to look at the butterfly, not the sludge pit. D) No, because the sludge pit is not a nuisance per se.

B) Yes, if Phillip knew young children had entered his land before and were likely to be injured.

A woman grew roses on her property. She went to the local gardening store and bought a bottle of plant food for her roses. The bottle was covered with pictures of beautiful roses and brightly colored lettering. The bottle contained ingredients beneficial to roses but harmful to humans if swallowed. The beautiful lettering did not contain warnings about harm to humans if swallowed. The woman lent the bottle to a friend who had three small children. One of the children saw the bottle on the counter and thought it contained perfume. The child sprinkled the bottle all over himself, swallowing some of the plant food. The child needed to be rushed to the hospital and have his stomach pumped. The mother of the child wants to sue the manufacturer and distributor of the plant food for her son's injuries. In the case against the store where the plant food was purchased, the mother will: A) win, since the store sold a dangerous product. B) win, since there was no warning about the potential danger if swallowed. C) lose, since she did not purchase the plant food. D) lose, since she should have better supervised her child.

B) win, since there was no warning about the potential danger if swallowed.

Owner subdivided his land into 15 lots, recording a plat of the subdivision (named "Maple Oaks") with no restrictions on any lot. Soon after, Owner sold Lot 1 to Buyer, via a deed that simply described the interest conveyed as a "fee simple absolute." Due to an economic downturn, no further lots were sold for over a year. During that time, in an attempt to boost its attractiveness to remote workers, City enacted a zoning ordinance allowing professional offices in areas zoned residential (reversing its previous ordinance, which prohibited such uses). Thereafter, Owner sold Lots 2 through 14. The deeds for Lots 2 through 14 included this language: "It is a condition of this conveyance and a covenant that shall run with the land for the benefit of each and all of the Lots within Maple Oaks Subdivision, City, State, that the property hereby conveyed shall be used solely as a single-family residence." Buyer was aware of this language in the deeds to Lots 2 through 14. After Lot 14 was sold, Buyer sold Lot 1 to Accountant via a deed that simply described the interest conveyed as a "fee simple absolute." Accountant filed an application for a building permit, indicating that the structure would

C) Accountant, because Lot 1 was conveyed first by Owner to Buyer without restriction and then by Buyer to Accountant without restriction.

Shortly after midnight, Dexter forced open the skylight of an expensive suburban residence in order to steal jewelry the occupants were reputed to have. When he had searched the entire house, he discovered that the residents had taken their jewelry on vacation with them. Dexter then noticed that a built-in security system installed by the owners had filmed him as he moved about the house. Dexter positioned a wooden bookcase under the camera, stacked some newspapers on it, set the papers afire, and then left the premises. By the time the fire department arrived and doused the flames, summoned by the house's security system, the bookcase was burned to cinders, and the wall and ceiling near the security camera were charred and peeling. The camera and its film were unharmed, and Dexter was subsequently apprehended and charged with common law burglary and arson. What should be the result? A) Dexter is guilty of burglary but not arson. B) Dexter is guilty of arson but not burglary. C) Dexter is guilty of both burglary and arson. D) Dexter is not guilty of either of the charged crimes.

C) Dexter is guilty of both burglary and arson.

Michael was arrested on a charge of credit card fraud. Chester County in the State of Oz presented the case to the grand jury, which "no billed" the charges because the federal government refused to allow a key witness that is in the witness protection program to come and testify because they believe that Chester has faulty security. However, a week later, when Morris County, which is also located in the state of Oz, presented the case to their grand jury, the grand jury handed down an indictment. When Michael is tried in the Morris County superior court, he is convicted of the charge. Michael appeals his conviction, claiming that his conviction in Morris County violates double jeopardy. What is the likely result? A) Double jeopardy applies because the Chester County grand jury "no billed" the charge. B) Double jeopardy applies because Morris County presented the same evidence to their grand jury as Chester County. C) Double jeopardy does not apply because it does not attach to grand jury proceedings. D) Double jeopardy does not apply because Chester County and Morris County are separate sovereigns.

C) Double jeopardy does not apply because it does not attach to grand jury proceedings.

A man took his 1975 Ford Fairlane to the auto repair shop owned by a mechanic in order to have a rebuilt water pump installed. It was necessary to leave the car in the shop overnight. Later that evening, unknown to the mechanic, the man was struck by a hit-and-run driver while in a marked pedestrian crossing and was severely injured, lapsing into a coma in the hospital where he was taken for treatment. The jurisdiction by statute permits an auto repair business to sell any car left for repairs beyond 30 days in order to recover the costs of repair. The mechanic had expended $120 in labor and $60 in parts in repairing the man's Fairlane.Twenty-eight days after the man had left his Fairlane for repair, a collector offered the mechanic $500 for the car. The mechanic agreed, even though he knew that 30 days had not elapsed since the car had been brought into the shop.Larceny, embezzlement, and false pretenses are separate crimes in the jurisdiction. Assume for purposes of this question only that the day after the mechanic sold the Fairlane to the collector, the man returned, offered to pay the repair bill, and demanded his car. The mechanic stated that the car had inadvertently been sold to

C) Embezzlement.

Ever since Zack was eight years old, he has been an avid model airplane builder and collector. He is now seventeen and has been buying and selling model airplanes for the past five years. Ben, a forty-five-year-old security guard who had never been interested in model airplanes before, saw Zack flying a radio-controlled model airplane one Sunday in a park and decided that he would like to own one. Ben asked Zack if he knew where he could get such a model, and Zack showed Ben an airplane he had in the back of his Land Rover. Although the airplane had a crack in its wing, when Ben asked Zack whether the crack would affect the airplane's flight, Zack replied that it would have no effect at all. Actually, Zack knew that the crack probably would interfere substantially with the airplane's ability to rise, sink or turn in response to the radio signals. Unaware of the significance of the crack, Ben arranged to come by Zack's house the following Saturday to bring Zack $150 cash and to pick up the model airplane. Zack wrote the terms of their agreement on a scrap of paper he found in his front seat, and both parties signed it. A few days later, Ben found out from a co-worker that the cracked wing

C) No, because Ben relied on a material misrepresentation.

A brother and sister lived in separate parts of the same large house that they had inherited from their parents. The brother and sister were both published songwriters. The sister found out one day that the brother had been traveling the country, playing the sister's songs, selling copies of them on CD and telling everyone that they were his songs. The sister filed suit against the brother in federal district court, alleging federal copyright violations. The sister was very angry with the brother, but she did meet with him at a local bar and handed him a summons and copy of the complaint. The brother filed a motion to dismiss, with the single argument that the sister had failed to state a claim upon which relief could be granted.If the brother later files an answer containing the objection that there was insufficient service of process, will he prevail? A) Yes, because service of process may not be affected by a person who is a party to the suit. B) Yes, because he was not served with process while at his residence. C) No, because he failed to include the objection in the original motion to dismiss. D) No, because he clearly indicated that he was aware of the suit.

C) No, because he failed to include the objection in the original motion to dismiss.

On October 1, the following notice was posted in the employees' lounge of the Lawrence Lumber Company: "The Company offers to any employee of the Lawrence Lumber Company who wins the annual Blue Ox Lumberjack Competition the additional prize of $1,000. All registration forms must be submitted through the personnel department before November 30."The Blue Ox Lumberjack Competition was conducted by an outside organization, unconnected with the Lawrence Lumber Company. The competition pits lumber workers from the Pacific Northwest against each other in traditional lumbering skills, such as log rolling and tree felling.Paul read this notice on October 3, and thereupon decided to intensify his effort to win the competition. He resolved to give up his weekend parties, and also began to exercise and eat more balanced meals. Paul left a note on the Personnel Director's desk saying, "I accept the company's $1,000 Blue Ox offer." This note was inadvertently placed in Paul's personnel file and never reached the Personnel Director or anyone else in a management position with the Company. On October 20, the above notice was removed and the following notice was substituted: "The Company regrets that ou

C) No, because of Paul's reliance prior to October 20 on the offer. An offer for a unilateral contract cannot be revoked once the offeree has made a substantial start on performance of the requested act. Here, Paul "intensified his effort" after reading the notice, thereby making a substantial start on performance. As a result, the offer was not effectively revoked as to Paul. Under most circumstances, an offer is revocable at any time before acceptance. However, an offer for a unilateral contract, acceptable by performance, becomes irrevocable when an offeree makes a substantial start on performance, as Paul did here. A reasonable lapse of time is one way an offer may be revoked, not how an offer becomes irrevocable.

A grantor executed a warranty deed conveying his land to a college, which contained the following language of conveyance: "to the college and its successors forever, so long as the land is used for college purposes." The college built a classroom building and a dormitory on the land and operated there for decades. The college decided to relocate and entered into a valid contract with a developer to sell the property for $20,000,000. The developer wanted to convert the existing structures into a condominium development and objected to the college's title. The jurisdiction has no statute on point, and the contract is silent on the quality of the title that the college is bound to convey to the developer. When the developer refused to close, the college sued the developer for specific performance and properly added the grantor as a party in interest to the litigation.Will the college prevail? A) Yes, because a charitable trust to support college purposes will attach to the proceeds of the sale to the developer. B) Yes, because the grantor cannot alter his warranty deed to the college. C) No, because the grantor's interest prevents the college's title from being marketable. D) No, because th

C) No, because the grantor's interest prevents the college's title from being marketable.

A lender loaned a borrower $200,000 to remodel his home. The borrower executed a promissory note secured by a mortgage on the home. The note required the borrower to repay the loan over a 10-year period. The lender promptly recorded the mortgage, which (at the time) was the only lien on the home. Several months later, the borrower borrowed $50,000 from his friend in order to purchase a new truck and gave his friend a mortgage on the home to secure repayment of the debt. The friend promptly recorded the mortgage.The borrower later lost his job and was struggling to make loan payments to both the lender and his friend. To accommodate the borrower's financial situation, the lender extended the amortization period of its loan to substantially reduce the amount of each monthly payment. The lender did not alter the interest rate or increase the principal amount of the loan. At the time of the modification of the lender's loan, the borrower was not in default in his payments to his friend. Neither the lender nor the borrower informed the friend of the modification of the lender's loan.Later, after the borrower missed six payments, the friend began a foreclosure action against the borrower. Duri

C) No, because the lender's modification of its loan was not detrimental to the rights of the friend.

The plaintiff, a corporation, sued the defendant in federal district court for defamation. As part of negotiations going into trial, the defendant agreed to submit to at least an 8-4 jury decision against the defendant as long as the defendant was able to conduct depositions of the plaintiff's employees of the defendant's choosing. At trial, 12 jurors were seated, and eight jurors voted in favor of the plaintiff, finding that the defendant was liable for defamation.If the defendant now argues that the judge may not enter a judgment against the defendant based on this non-unanimous vote by the jury, will the defendant prevail? A) Yes, because a jury verdict in a civil trial requires at least 9 jurors to vote in favor of a plaintiff. B) Yes, because a jury verdict in a civil trial must be unanimous. C) No, because the parties were free to stipulate to accepting a non-unanimous verdict D) No, because unanimous jury verdicts are only required in certain cases.

C) No, because the parties were free to stipulate to accepting a non-unanimous verdict

A plaintiff was hurt when they were hit by a cart in a grocery store. The plaintiff properly commenced an action against the owner of the store in the appropriate U.S. District Court. A verdict was returned in favor of the plaintiff for $500,000. The owner of the store moved for a new trial. The trial judge agreed to grant a new trial unless the plaintiff accepted a reduction in the verdict from $500,000 to $50,000. The plaintiff agreed to the reduction under protest and then appealed. Plaintiff asserted as error the trial court's order granting a new trial unless they accepted a reduction in the verdict from $500,000 to $50,000.Will the plaintiff's appeal from the order of remittitur be successful? A) Yes, because remittitur is prohibited in federal courts. B) Yes, because the court's order violated the student's Seventh Amendment right to a trial by jury upon the issue of damages. C) No, because the plaintiff elected to accept the remittitur. D) No, because the Seventh Amendment does not apply to damages issues.

C) No, because the plaintiff elected to accept the remittitur.

A lawyer was overjoyed when she received the announcement from the Committee of Bar Examiners declaring that she had passed the bar exam. However, she was even more pleased when a law firm offered her an associate position. On December 1, the lawyer signed a written one-year employment contract with the firm. The contract provided for a $48,000 salary, with the one-year term commencing January 1.The lawyer failed to appear for work on January 1 due to injuries sustained in an automobile accident. The firm learned that the lawyer had been injured in an automobile accident and could not start work until February 1. The firm hired an associate, the only available substitute, to fill the position until February 1. The associate demanded, and the firm paid the associate, a salary of $4,500 for the month of January. May the firm recover from the lawyer the additional $500 expense for the January employment which the firm incurred? A) Yes, because the lawyer could reasonably foresee that the firm would have to hire a substitute. B) Yes, because the lawyer's promise to work for the firm beginning January 1 was an unconditional promise. C) No, because this is a personal service contract where the

C) No, because this is a personal service contract where the lawyer's illness excused her failure to perform.

One night, after Drake had been in California for about six months, he missed his girlfriend in Texas so badly that he wanted to leave California immediately. Lacking cash for the trip, Drake approached his friend Sam for a loan. Sam declined. Drake then went to his neighbor, Hugh, and offered to buy Hugh's used stove for $50. Drake wrote Hugh a check for $50 for the stove. Drake then sold the stove to Sam for $50 and left for Texas. Having discovered that Drake's account was closed and that Sam had his stove, Hugh sued Sam for conversion. Which of the following facts, if true, would give Hugh the best chance of recovery against Sam? A) Drake knew his account was closed at the time he wrote the check to Hugh. B) Sam knew Drake had closed his bank account. C) Sam knew Drake had given Hugh a check not covered by sufficient funds. D) Sam intended to make a gift of the stove to his sister.

C) Sam knew Drake had given Hugh a check not covered by sufficient funds.

Stacy had a pet poodle for four years. The dog was her whole life. She took it wherever she went and gave it as much attention as a child. When Stacy returned from work, she found the dog locked in a back bedroom. The maintenance man was in the apartment and told Stacy he came to fix the plumbing when the dog grabbed onto the man's pants and bit through the leg of the pants. Stacy was astonished and assured the man that the dog must have been very upset to do so.Four days later, Stacy's sister in law arrived for a visit with her two children. The children were young and not used to dogs. When the little boy approached the dog and yelled, "Doggie!", the poodle jumped on him and bit his ear.If the little boy wants to sue Stacy for strict liability, what must he prove to succeed? A) That Stacy was negligent in not leashing the dog. B) That the boy yelling at the dog did not contribute to his injury. C) That Stacy knew the dog could bite the boy. D) That the boy and sister in law were invited guests of Stacy.

C) That Stacy knew the dog could bite the boy.

A seller conveyed residential land to a buyer. The seller gave the buyer a warranty deed that recited the full purchase price and that contained no exceptions. The buyer was able to borrow 80% of the purchase price from a bank, but only had enough funds to cover half of the shortfall (10% of the purchase price).The seller agreed to finance the remaining 10% of the purchase price for the buyer, for which the buyer signed an unsecured promissory note at closing. At closing the buyer also signed a promissory note to the bank for 80% of the purchase price, secured by a mortgage on the land. The bank was aware of the promissory note given by the buyer to the seller.The buyer has defaulted on both loans, and the seller has come to you for advice. There are no applicable statutes. Whose loan has priority? A) The seller's, because a promissory note to a seller has priority over a bank loan for residential real estate. B) The seller's, because the bank was aware of the seller's loan, giving it an equitable priority. C) The bank's, because it was secured by a purchase-money mortgage. D) The bank's, because the seller can finance a part of the purchase price only by using an installment land contra

C) The bank's, because it was secured by a purchase-money mortgage.

A seller and a buyer executed a written contract for the sale of identified acreage. The terms of the contract stated that the purchase price was $225,000 due at the time of closing. At the time they executed the contract, the seller and the buyer had decided not to include a specific closing date, as the buyer was not certain how he would obtain financing for the purchase price.Ten days later, a developer offered the seller $300,000 for the acreage. When the seller asked the buyer if they could agree to cancel the contract, the buyer refused. The seller then refused to complete the transaction, telling the buyer that the contract was unenforceable under the statute of frauds because an essential element (time for performance) was not agreed upon by them and was not expressly stated in their written contract. Thereafter, the seller sold the acreage to the developer.If the buyer sues the seller for breach of contract, who is likely to prevail? A) The seller, because the contract is unenforceable under the statute of frauds. B) The seller, because the time for performance is presumed to be of the essence. C) The buyer, because the court will infer that performance within a reasonable time

C) The buyer, because the court will infer that performance within a reasonable time was intended by the parties.

Cruz had skied for many years. Jamison was a ski instructor, owner of her own ski school, and a member of the National Ski Patrol. Cruz was proceeding down a relatively steep hill on an advanced course by a series of traverses. He was crossing the slope a number of times to diminish the angle of descent—the skiing equivalent of switchbacks in hiking. Jamison came straight down the hill, saw Cruz midway across the hill in one of his traverses, and attempted to ski behind Cruz. Jamison miscalculated the speed at which Cruz was traveling, and ran directly into him, fracturing Cruz's knee. Jamison's conduct will be judged against: A) The conduct of a reasonably prudent person. B) The conduct of a reasonably prudent person in an emergency. C) The conduct of a reasonably prudent person with superior skiing knowledge and expertise. D) The conduct of a reasonably prudent person with the same age, knowledge, and experience.

C) The conduct of a reasonably prudent person with superior skiing knowledge and expertise.

Just over six years ago, a landlord and a tenant entered into a five-year lease for a commercial building. Under the terms of the lease, rent was payable on the first day of the month and, in the event of a default by the tenant, the landlord had the right to terminate the lease.The tenant remains in possession of the building and continues to pay rent according to the lease terms, which the landlord has accepted. The fair rental value of the building is now significantly more than what the tenant is paying. A business recently contacted the landlord, offering to rent the building at the current fair rental value.Six weeks ago, when the tenant made the usual rent payment, the landlord accepted the payment and informed the tenant in writing that the tenant had until the end of the month to vacate the premises.The tenant remains in possession of the building and does not want to relocate. The tenant mailed next month's rent payment (in the amount of the current fair market value) to the landlord early, together with a letter offering to pay the landlord the current fair rental value.Who is entitled to possession? A) The tenant, because the landlord has not shown good cause to terminate the

C) The landlord, because the landlord gave timely notice of termination.

An owner of land and his neighbor owned adjacent homes in a subdivision. Several homeowners in the subdivision had built detached storage sheds in their backyards. The restrictions recorded for the subdivision plat did not prohibit such sheds. The owner and the neighbor, however, thought the sheds were (in the neighbor's words) "an eyesore," and they agreed in writing not to build or place detached storage sheds in their respective yards. Their agreement, which they recorded, stated that it was enforceable by and against their respective heirs, assignees, and successors in interest.Some years later, the neighbor died and his niece inherited his home. Not long after the niece moved into the home, she learned of the restriction from the owner. She told the owner that she never signed anything and would be putting a detached storage shed in her backyard.If the owner sues to prevent the niece from placing a shed in her yard, who will prevail? A) The niece, because several homeowners in the subdivision have sheds in their yards. B) The niece, because the niece did not consent to the restriction. C) The owner, because the restriction is binding on the niece as a successor of the neighbor. D) T

C) The owner, because the restriction is binding on the niece as a successor of the neighbor.

A woman borrowed $750,000 from a bank, giving a mortgage on a commercial building she owned as security. The mortgage contained this provision: "The borrower agrees that prepayment may be made on this loan in the amount of the borrower's choosing but only if accompanied by a prepayment fee in the amount of 5% of the amount prepaid, except that no prepayment shall be made on this loan during the first two years from the date of execution of this mortgage."After making payments for nearly a year, the woman received an unexpected inheritance of $1,000,000 million and decided to pay off the remaining balance due. Concerned that the bank might refuse prepayment or at least insist on payment of the 5% prepayment fee, the woman seeks your legal advice regarding the enforceability of the quoted language.In a jurisdiction with no applicable statute, what advice should you give her? A) The prepayment ban is unenforceable, but the prepayment fee is enforceable. B) The prepayment ban is enforceable, but the prepayment fee is unenforceable. C) The prepayment ban and the prepayment fee are both enforceable. D) The entire clause is unenforceable, because it violates a public policy favoring the prompt

C) The prepayment ban and the prepayment fee are both enforceable. In the context of commercial real estate, both the prepayment ban and the prepayment fee are valid and enforceable. While there is a public policy favoring the prompt and early repayment of debt, that public policy applies to residential real estate, not commercial real estate

Able knew that Charlie possessed an expensive Rolex watch. Knowing that Charlie did not wear his Rolex when he went to the gym, Able decided to break into Charlie's home while Charlie was at the gym and steal his Rolex.After observing Charlie leave home carrying his gym bag, Able broke into Charlie's home and began looking for the Rolex. Charlie then returned home unexpectedly because he had forgotten his cellphone. Able knocked Charlie to the floor and then tied him up with a rope he found in the garage. Able then located the Rolex in Charlie's bedroom and left with it in his possession. After Able had left, Charlie choked to death from the ropes while trying to free himself.Able was arrested and charged with robbery, but he was acquitted. Thereafter, Able was rearrested and charged with felony murder, with the underlying felony being robbery. With respect to Able's criminal liability for felony murder, which of the following is most correct? A) The robbery acquittal precludes any subsequent prosecution under the doctrine of res judicata. B) The robbery acquittal precludes any subsequent prosecution under the doctrine of collateral estoppel. C) The robbery acquittal precludes any subseq

C) The robbery acquittal precludes any subsequent prosecution under the doctrine of double jeopardy.

A man was prosecuted for violation of a state statute which defines as a felony "the taking or accepting of any property, money or services by a state driver's license examiner from or on behalf of an examinee being tested for driving proficiency." Testimony at trial established that the man was taking his driving test from an examiner, a state driver's license examiner, when he offered the examiner $1,000 if the examiner would overlook an illegal lane change the man had just made (and which would disqualify him from obtaining a driver's license). The examiner accepted the $1,000, and was subsequently convicted of violating the same statute for which the man was being prosecuted as an aider and abettor. The man's best argument for a dismissal of the charge against him is that: A) Only a driver's license examiner can commit the crime defined by the subject statute. B) He cannot be convicted of committing a crime as to which he is the victim. C) The statute is so defined as to indicate that the legislature intended only the recipient of the property, money or services to be punished. D) He did not assist the examiner in violating the subject statute.

C) The statute is so defined as to indicate that the legislature intended only the recipient of the property, money or services to be punished.

Potter and Daphne entered into a written contract whereby Potter agreed to manufacture for Daphne, and Daphne agreed to buy, 100 crates of matches at a price of $75 per crate. The contract specified that, "it is expressly agreed that Daphne shall be under no obligation under this contract unless 100 crates of matches are delivered to her at her place of business no later than November 1." Eighty crates of matches, meeting Daphne's specifications, were tendered by Potter on November 1. The remaining twenty crates of matches were tendered on November 5. Daphne refused to accept any of the crates of matches.In an action by Potter against Daphne for breach of contract, which of the following would be Daphne's best defense? A) Between the time the contract was made and November 1, the market price of matches dropped 80 percent. B) As of November 1, Potter had not substantially performed. C) Time was of the essence. D) An earthquake had completely destroyed Daphne's place of business on November 1.

C) Time was of the essence.

The publisher of a magazine announced that Poet One was the winner of a $10,000 prize for the best haiku of the month. Before the publisher awarded the prize to Poet One, it received a letter from Poet Two claiming that Poet Two had written the haiku, and that Poet One's entry was stolen from either Poet Two or Poet Two's agent. The publisher, who is a citizen of California, brought a statutory interpleader action in the appropriate U.S. District Court in California seeking a declaratory judgment as to which of them was entitled to the prize money. Poet One is a citizen of Montana, and Poet Two and Poet Two's agent are both citizens of Nebraska.Which of the following statements is not accurate? A) The federal court has subject-matter jurisdiction over this action. B) Poet Two may not file a common law misrepresentation claim in this interpleader action against her agent seeking $10,000 from her in the event that the prize money is awarded to Poet One. C) Venue is proper in California under statutory interpleader. D) The jurisdictional requirements for statutory interpleader are met.

C) Venue is proper in California under statutory interpleader.

A seller contracted in writing to sell land improved with a vacant warehouse to a buyer. The contract was silent as to the risk of loss in the event of damage to or destruction of the warehouse building between contract signing and closing, the duty of either party to carry insurance, or contingencies. The day after the contract was signed, the seller canceled his insurance on the property. The buyer was unaware of the seller's actions and did not take out a policy of insurance on the property. Soon thereafter (several weeks before the closing date), a tornado struck and destroyed the warehouse. The jurisdiction has no applicable statute. The buyer sues the seller to recover her earnest money, claiming a right to cancel the contract. The seller claims that the buyer must perform because the risk of loss passed to the buyer before the tornado.If the court holds for the seller, what is the most likely reason for the court to do so? A) The buyer's constructive possession after the execution of the contract created an affirmative duty to protect the property against loss. B) The seller's cancellation of insurance coverage transferred the risk of loss to the buyer. C) When the contract was si

C) When the contract was signed, the buyer became the owner of the land under the doctrine of equitable conversion.

On October 1, the following notice was posted in the employees' lounge of the Lawrence Lumber Company: "The Company offers to any employee of the Lawrence Lumber Company who wins the annual Blue Ox Lumberjack Competition the additional prize of $1,000. All registration forms must be submitted through the personnel department before November 30." The Blue Ox Lumberjack Competition was conducted by an outside organization unconnected with the Lawrence Lumber Company. The competition pits lumber workers from the Pacific Northwest against each other in traditional lumbering skills, such as log rolling and tree felling. Paul read this notice on October 3, and thereupon decided to intensify his effort to win the competition. He resolved to give up his weekend parties, and also began to exercise and eat more balanced meals. Paul left a note on the Personnel Director's desk saying, "I accept the company's $1,000 Blue Ox offer." This note was inadvertently placed in Paul's personnel file and never reached the Personnel Director or anyone else in a management position with the Company. On October 20, the above notice was removed, and the following notice was substituted: "The Company regrets tha

C) contractual offer, creating a power of acceptance.

The state government advertised for bids from general contractors on construction of a state office building of specified dimensions. A contractor, who intended to submit a bid on the project, solicited bids from various sub-contractors for framing, electrical work, plumbing, etc. The contractor subsequently received a sub-bid on the plumbing work for the project of $510,000 from a plumbing subcontractor. The subcontractor's bid was $10,000 lower than any competing sub-bid. After compiling all the acceptable sub-bids, the contractor submitted to the state government a bid on the total project for $5 million. This was the lowest bid submitted to the state government, which accepted the contractor's bid on that basis. That same day, the contractor notified the subcontractor that his plumbing sub-bid was accepted. A week after the state government accepted the contractor's bid, the subcontractor notified the contractor that an accounting error had resulted in his plumbing sub-bid being too low by $50,000; the actual bid should have been $560,000 for the plumbing work.If the subcontractor brings an appropriate action to rescind his contract with the contractor for the plumbing work on the st

C) the contractor, because he was unaware of the subcontractor's mistake and had no reason to know of it.

A construction company was building a country club, including extensive recreational facilities, for a group of wealthy investors. The project had been underway for eight months and involved considerable use of heavy equipment. For the past two months, during the long summer evenings, children from a nearby residential development had been slipping through a huge rip in the chain link fence that surrounded the construction site and playing on the heavy equipment. A 13-year-old boy was able to start one of the bulldozers and drove it around the construction site. As he neared the pit dug in preparation for installation of the country club swimming pool, the boy attempted to turn away, but the steering mechanism of the bulldozer failed, and the boy was unable to prevent it from toppling into the pit. The boy leaped clear and was uninjured, but a six-year-old child, who had been playing in the swimming pool excavation, was seriously injured when the bulldozer fell onto him.If the six-year-old child, through a legal representative, brings an action against the 13-year-old boy to recover for his injuries caused by the bulldozer accident, which of the following standards is most likely to be a

D) Because the boy was a defendant engaged in an adult activity, his actions will be judged by the same reasonable person standard applied to adults.

A tenant leased an apartment for $1,000 per month from a landlord for one year via a written lease. The tenant moved in and paid the rent for six months. The tenant then took a job in another city and moved. Without speaking to the landlord first, the tenant's cousin moved into the apartment under a written agreement with the tenant ,whereby the tenant gave her cousin "all of my rights under the lease" for the balance of the lease term. The cousin paid rent directly to the landlord for three months. For the last three months of the lease, neither the tenant nor the cousin paid rent. The cousin moved out with two months left on the lease. The landlord had been deployed as a member of the armed forces and did not learn about the vacancy or the default until after returning from deployment at the end of the lease term. The landlord is suing the tenant and the tenant's cousin, jointly and severally, seeking to recover the unpaid rent for the last three months of the lease.What is the likely outcome of the landlord's lawsuit? A) The cousin is liable for $2,000 based on privity of estate with the landlord, which lasted only until the cousin vacated the premises. The tenant is liable for $1,000

D) Both the tenant and the cousin are both liable for the full $3,000, because the tenant is in privity of contract with the landlord and the cousin is in privity of estate with the landlord as an assignee of the tenant.

The plaintiff was injured when struck by a softball that the defendant hit during a softball game at the park. The plaintiff sued the defendant in federal court based on diversity jurisdiction. Before trial, in compliance with Rule 26(a) of the Federal Rules of Civil Procedure, the plaintiff disclosed to the defendant a list of all witnesses to the accident, all documents the plaintiff had in their possession, and a computation of damages from the injury. A short time later, the plaintiff learned of another witness to the accident. The witness had left on an international trip shortly after the accident, and had not been part of the police officer's accident report. In fact, the only way the plaintiff learned of him was because the witness saw something about the case on television and called the plaintiff. At trial, the plaintiff called the new witness to testify as part of their claim, because this witness was the only one who had a complete and unbroken view of the incident. The defendant argued that the witness should be excluded because the plaintiff failed to disclose the witness.How should the court rule? A) Allow the witness to testify, because the plaintiff provided the informat

D) Exclude the witness from testifying, because the plaintiff did not supplement their initial witness list.

A family owned a restaurant that had been in business for 60 years. Over that time, it had been the gathering spot for many of the politicians and celebrities who lived in the city. A new restaurant opened up right next to the family's restaurant. Soon after the opening of the new restaurant, the family's restaurant started losing a significant amount of business. The family filed a complaint in federal court claiming the "new restaurant illegally encroached on their historically recognized business territory." The family then started placing advertisements for their restaurant on the new restaurant's windows, and got their 16-year-old daughter to post bad reviews online about the new restaurant.Assuming the federal court has jurisdiction to hear the lawsuit, what should the new restaurant owner's family do in response to the complaint? A) File a motion for judgment as a matter of law. B) File a counterclaim regarding the man's signs and bad reviews. C) File an answer. D) File a motion for dismissal.

D) File a motion for dismissal.

Doris was a committed vegan, eating only plant-derived foods, and the thought of all the slaughter going on to provide meat for the less-enlightened members of society made her nauseous. One night, Doris decided to make meat consumers nauseous to try to encourage them to stop eating meat. She went to the butcher's section of the supermarket where she was employed as produce manager and sprinkled a nausea-inducing chemical on all the meat, fowl, and fish products. Despite being careful, Doris accidentally put too much powder in one package of hamburger, and when the woman who bought it served it to her family, her husband, who was unusually susceptible, died as a result of ingesting the chemical.In Doris's jurisdiction, murder is defined as: "the unlawful killing of a human being with malice aforethought. Such malice may be express or implied. It is express when there is manifested a deliberate intention unlawfully to take away the life of another person. It is implied when no considerable provocation appears or when the circumstances attending the killing show an abandoned and malignant heart. All murder which is perpetrated by willful, deliberate, or premeditated killing or which is com

D) Involuntary manslaughter, since she did not intend to kill anyone.

Susan and David, both experienced backpackers, were climbing rocky switchbacks when Susan lost her balance and fell twenty feet to a ledge below, breaking her leg. David immediately hiked to the nearest wilderness ranger station where a private helicopter service, Air Rescue, the only such service operating in the area, was contacted to airlift Susan to a hospital. The Air Rescue pilot and co-pilot lifted Susan from the ledge and placed her in the gurney which paralleled the helicopter cab. The Air Rescue pilot then directed Susan to sign a document stating that she agreed not to sue Air Rescue for any injuries she might incur as a result of the helicopter flight because of the public service and emergency nature of the work. Susan read and signed the document. The Air Rescue pilot then strapped one of the two safety straps on the gurney across Susan's chest. Upon arriving at the hospital, Susan complained of severe back pain. Her physician states that the trip in the helicopter caused the back injury.If Susan brings suit against Air Rescue for her back injury, what will be the effect of the document she signed? A) It is an express assumption of risk which will bar Susan's suit against A

D) It will not bar Susan's recovery because Air Rescue is the only air lift service available in the area. An express assumption of risk, a disclaimer clause, is valid only if three criteria are met: 1) the plaintiff is aware of its terms, 2) the injury that occurs is within the risks of which the plaintiff agreed to relieve the defendant, and 3) the disclaimer is not contrary to public policy.

A sailor was out in his small sailboat in the channel between the shore and an offshore island when he negligently caused his boat to become swamped and sink. The sailor was thrown into the ocean, but was held afloat by his flotation jacket. About thirty minutes later, a yachtsman came along in his yacht on his way to a party on the island. He saw the sailor in the water and stayed long enough to determine that the sailor was not injured. "Someone else is bound to come along, and I'm late for my party. I'll advise the Coast Guard when I reach the island," said the yachtsman as he sailed away without picking up the sailor. About 30 minutes later, the yachtsman reached the island and notified the authorities of the sailor's position. When the Coast Guard reached the sailor, they discovered that he had been run over and severely injured by a powerful ocean racing boat operated by a captain. The captain had seen the sailor in time to avoid hitting him, but was sufficiently intoxicated so that he was unable to steer the boat aside quickly enough.The jurisdiction in which this accident occurred has not adopted any form of comparative negligence. The sailor brought an appropriate action against

D) Judgment for the sailor, because the sailor was in helpless peril.

Manson and Brown liked to travel to a remote vacant field and practice shooting with their revolvers. They would usually set up discarded cans and bottles and have contests to see who could be the most accurate or who could shoot the quickest, "Old West" style—drawing from a holster and shooting from the hip without bringing the pistol to eye level. After one such contest, while Brown was setting up some more cans at which to shoot, Manson stayed where he was and reloaded his pistol. When Brown turned to return to where Manson stood, Manson shouted, "Hey Brown, why don't you dance for me!" Manson then drew his pistol and quickly fired several shots at Brown's feet, aiming each time to miss striking any part of Brown. However, one of the bullets ricocheted off the rocky ground and struck Brown in the eye, killing him.What is the most serious crime with which Manson could be convicted? A) No crime, because the killing was clearly accidental. B) Involuntary manslaughter, because Manson did not intend to kill or injure Brown. C) Voluntary manslaughter, because Manson did not intend to kill or injure Brown. D) Murder, because Manson acted with malice.

D) Murder, because Manson acted with malice.

A car collector discovered that a seller was offering to sell a rare vintage automobile which the car collector had long sought. The seller agreed in writing to sell the vintage automobile to the collector for $25,000. According to the terms of their written agreement, the collector was to tender payment to the seller on the day that she comes to pick up the vintage automobile at the seller's garage. On the day before the collector was due to pick up the vintage automobile, a tornado destroyed the seller's garage and its contents, including the vintage automobile. At the time the vintage automobile was destroyed, the risk of loss remained on the seller. Shortly afterwards, the collector discovers that a vintage automobile identical to the one the collector had contracted to purchase from the seller was recently sold at auction for $40,000.The seller sues the collector to recover the contract price of $25,000. The collector counter-sues the seller for $40,000. Who should recover on her claim? A) Only the seller should recover on her claim. B) Only the collector should recover on her claim. C) Both the collector and the seller should recover. D) Neither the collector nor the seller should

D) Neither the collector nor the seller should recover.

Detective Johnson had been investigating a case of missing college students for a year and a half, but had been unsuccessful at solving the disappearance. Johnson suspected that Drake was involved in the disappearance, but could not prove it. He had asked for permission to search Drake's house, but he was refused.One day, while Johnson was staking out Drake's house, a furnace repair truck pulled up. Drake had requested the repairman come to his house and repair his furnace. When the repairman got out of his truck to get some parts, Johnson explained the situation and asked the repairman to look around a little bit for anything that could have belonged to the missing students. The repairman entered the house and, in looking in a container in the basement, found several college student IDs. He stuffed the IDs in his toolbox and left. He then met Johnson around the corner and turned over the items to Johnson. The IDs are identified as belonging to the missing students.Johnson is arrested and charged with homicide of the missing students. At trial, the state moves to introduce the IDs into evidence but the defense objects. What should be the result? A) The IDs should be admissible because th

D) The IDs should be inadmissible.

A landlord leased an apartment to a tenant by a written lease that provided: "the landlord may enter the leased premises to inspect or repair the premises and its fixtures. When possible, advance notice shall be provided before entering."After discovering water coming through the ceiling of an apartment on the floor below the tenant's apartment, the landlord's building manager rushed to the tenant's apartment and used her pass key to open the door. The building manager discovered that the water line to the tenant's kitchen sink had broken. As she was shutting off the water to the broken line, the building manager encountered the tenant, who had been asleep when the building manager entered. The tenant ordered the building manager to get out of the apartment.The tenant has since changed the locks and refuses to let the building manager enter the apartment to complete the necessary repairs so that water service can be restored to the kitchen. The tenant has also reduced the amount of rent paid, claiming an offset for the lack of water in the apartment's kitchen.If the landlord sues to recover the lost rent and to compel the tenant to allow access for repairs, who is likely to prevail? A) T

D) The landlord, because the lease expressly provides the landlord with the right to enter the apartment to inspect and make repairs.

Three years ago, a lender was awarded a judgment against a borrower and thereafter filed the judgment in a county. One year ago, the borrower purchased land in the same county via warranty deed, which the borrower promptly recorded. Soon after, the borrower then borrowed $50,000 from his cousin, giving a mortgage secured by the land. The cousin failed to make a title search before making the loan to the borrower but did promptly record the mortgage. The borrower has made no payments either to the lender or to the cousin. The cousin is considering whether to foreclose on the mortgage and is seeking your legal advice. The jurisdiction has enacted the following statutes: "Any judgment of a court of this state shall be a lien on real property owned at the time of filing or acquired thereafter by any person against whom such judgment is entered for a period of 10 years from the date of filing." "No deed, conveyance, mortgage, or lien that has not been recorded shall have effect against subsequent purchasers for value who take without notice and who shall first record."Which lien takes priority? A) The cousin, because she holds a purchase-money mortgage. B) The cousin, because a judgment lien

D) The lender, because the lender's judgment was filed first.

A detective received information from a confidential informant, who stated that a man had an active gambling and loan sharking business. He indicated that he knew that the man laundered the money from the illegal activities through his restaurant but that no records or anything else incriminating was kept at the restaurant. The informant indicated that the man kept outstanding loan and betting records at his home, and that every Wednesday afternoon the man moved the settled loan and betting records in his car to a safe deposit box.With this information in hand, the detective set up surveillance outside of the man's house. Over the course of two weeks, he saw heavy traffic going in and out of the man's house, with particularly heavy traffic on Friday and Saturday, the days before college and professional football games. The detective then left on a two-week vacation. Upon his return, he applied for and was given a search warrant for the man's house.A team of officers executed the search warrant on the man's home on Wednesday afternoon. The team found numerous records of loan sharking and gambling. As a result, the man was arrested. At trial, the man moved to suppress the records found at

D) The records should be admitted.

Detective Johnson had been investigating a case of missing college students for a year and a half but had been unsuccessful at solving the disappearance. Johnson suspected that Drake was involved in the disappearance but could not prove it. He had asked for permission to search Drake's house, but he was refused. One day, while Johnson was staking out Drake's house, a furnace repair truck pulled up. Drake had requested that the repairman come to his house and repair his furnace. When the repairman got out of his truck to get some parts, Johnson explained the situation and asked the repairman to look around a little bit for anything that could have belonged to the missing students. The repairman entered the house and, in looking in a container in the basement, found several college student IDs. He stuffed the IDs in his toolbox and left. He then met Johnson around the corner and turned over the items to Johnson. The IDs are identified as belonging to the missing students. Based on the findings of the IDs, Johnson is granted a search warrant for Drake's house. In executing the search warrant, additional incriminating evidence against Drake is discovered. At trial, the defense challenges the

D) The search warrant is invalid. The evidence used to obtain the search warrant was obtained illegally. The state cannot circumvent the state action requirement by employing private citizens to do their work. In effect, the repairman is an agent of the state because he was acting on the behest of Detective Johnson and the activities he engaged in took him beyond the scope of his employment by Drake. Therefore the warrant is invalid.

A buyer purchased a 50-year-old house from a seller. Their contract required the seller to give the buyer a warranty deed at closing. The buyer asked the seller no questions regarding the condition of the house and their contract was silent on this point. At closing, the seller provided a warranty deed, which the buyer promptly recorded. About three weeks after the closing, in rapid succession the basement flooded, the central air conditioning stopped working, and the roof started leaking so badly and in so many places that no part of the second floor of the house could be occupied. The seller was genuinely surprised when told about the condition of the house.If the buyer sues the seller for damages, who will prevail? Assume that the jurisdiction has no applicable statute. A) The buyer, based on covenants of title contained in the deed. B) The buyer, because a warranty of fitness is implied with a conveyance of residential real property. C) The seller, because of the doctrine of merger. D) The seller, because the seller gave no warranty regarding the condition of the house.

D) The seller, because the seller gave no warranty regarding the condition of the house.

Smith is a special summertime assistant to the local police department. Smith desired to become a member of the police department, but he would be required to attend a local police mini-academy, which he has not yet done. One day, as he was assisting with crowd control at a local charity event, a car accident occurred on one of the side streets nearby. Smith ran to help the accident victims. He attended to Sally, who had just gotten out of her car, fallen to the ground, and started crying hysterically. Smith found this curious because the accident didn't seem so bad. He told Sally to calm down and that everything would be all right. Sally responded, "It will never be alright again. It's all my fault! I overreacted and now I can't take it back!" As she finished this sentence, the EMTs arrived and started to check on Sally. Smith returned to his crowd control duties.Sally is charged with reckless driving. At trial she moves to suppress her statement to Smith. What is the likely outcome of this motion? A) The statement will be deemed inadmissible due to the fact that Sally was not given her Miranda warnings. B) The statement will be deemed inadmissible due to the fact that Sally had just be

D) The statement will be deemed admissible because Sally was not in custody at the time of the statement.

A woman entered into a written contract by which a construction firm agreed to construct a daycare center on the woman's land within ten months in exchange for $100,000. During construction of the daycare center, a dispute between Canadian and American timber interests increased the cost of lumber significantly. The construction firm discovered that its costs for lumber to complete the daycare center would be $20,000 higher than it had expected. The woman and the construction firm subsequently executed a written modification of the contract which provided that the square footage of the daycare center would be reduced from 2,000 to 1,800 square feet, and that the construction firm would still be entitled to the full contract price of $100,000. When the construction was completed, the woman refused to pay $100,000 as demanded by the construction firm, claiming that the ten percent reduction in the size of the daycare center was a breach of the original contract entitling her to damages reducing the amount owed on the contract.In an appropriate action to determine the rights of the parties, applying traditional rules of contract law unmodified by statute, who should prevail? A) The construc

D) The woman, because the written agreement reducing the size of the daycare center was not supported by consideration.

An American tourist from State B was walking in Italy when she was hit by a drunk driver who was also an American citizen from State A. The tourist suffered serious injuries and was in a hospital recovering from her injuries for seven weeks. When she was released from the hospital, she returned home and sued the driver in federal court in State B, based on diversity jurisdiction. The driver seeks dismissal of the action based on the doctrine of forum non conveniens.When will a federal court most likely grant such a motion? A) When the plaintiff's choice of forum is unfairly prejudicial to the defendant. B) Only if the plaintiff does not object. C) When the court can transfer the case to another federal court. D) When the convenience of the parties and witnesses, the ease of access to proof, calendar congestion, laws that apply, other factors favor granting the motion, and the court cannot transfer the case to another federal court.

D) When the convenience of the parties and witnesses, the ease of access to proof, calendar congestion, laws that apply, other factors favor granting the motion, and the court cannot transfer the case to another federal court.

A carpenter obtained a favorable jury verdict against a homeowner in a trial for breach of contract in federal court. The homeowner made a motion for judgment as a matter of law at the close of the carpenter's case and again at the close of all the evidence. The court denied both motions. During the trial, the court erroneously refused to admit into evidence the testimony of someone who was with the homeowner on the day that the contract was signed. That error did not affect the homeowner's substantial rights. The homeowner filed a timely renewed motion for judgment as a matter of law after the jury's verdict and in the alternative filed for a motion for a new trial based upon the erroneous exclusion of evidence. The trial court decided to grant the renewed motion for judgment as a matter of law.Must the trial court deny the alternative motion for a new trial? A) No, because the court granted the renewed motion for judgment as a matter of law B) No, because the decision to exclude the witness from evidence was erroneous. C) Yes, because the court granted the renewed motion for judgment as a matter of law. D) Yes, because the builder's substantial rights were not affected by the erroneous

D) Yes, because the builder's substantial rights were not affected by the erroneous exclusion of evidence.

A State A creditor made an unsecured $80,000 loan to a woman in State B, who became delinquent in her payments. On February 1, the creditor filed suit against the woman in federal court in State B, and sought to serve the defendant personally in State B so as to establish personal jurisdiction over her. The defendant was never served in State B. On February 15, the creditor learned that the defendant had moved to State C to begin a new job and arranged to have the defendant personally served with a summons at the defendant's home in State C. On May 1, the defendant appeared in State B court for the first court date and told the court that, although she had been traveling back and forth between an apartment in State B and her new home in State C, she now lived exclusively in State C. The defendant added that she began her efforts to relocate to State C well before the creditor filed the lawsuit, but did not move there until after the lawsuit was filed. The defendant argued that, because she is no longer domiciled in State B, the court in State B does not have personal jurisdiction.Does the federal court in State B court have personal jurisdiction over the defendant? A) No, because the cre

D) Yes, because the defendant was domiciled in State B when the incidents giving rise to the suit occurred and the lawsuit was filed.

A builder and an owner entered into a written contract under which the builder agreed to build a room addition onto the owner's home in exchange for $20,000. The $20,000 was to be paid in three installments, $5,000 at the beginning of the project, $5,000 when the addition was one-half completed, and $10,000 upon completion. The contract contains a clause prohibiting assignment of rights. However, in the midst of construction, the builder received a letter from his lumber supplier demanding payment of a large, overdue bill. The builder assigned the right to the final contract payment of $10,000 to the lumber supplier, who was unaware of the provision in the contract prohibiting assignment. The owner made the first two payments required by the contract as scheduled, but she defaulted on the final payment.If the lumber supplier files suit against the owner, will the lumber supplier prevail? A) No, because a clause in the contract prohibited assignment. B) No, because the assignment was not in writing. C) Yes, if the owner consented to the assignment. D) Yes, with or without the owner's consent to the assignment.

D) Yes, with or without the owner's consent to the assignment.

A painter agreed to paint two cottages. The owner of the cottages agreed to pay the painter $1000. The agreement did not provide for the time of payment.On the day the painter arrived to begin painting the cottages, the painter demanded full payment at that time. The owner refused to make payment in advance. If the painter sues the owner for breach of contract, the court should find for: A) the painter, because where an agreement is silent about the order of performance, payment is required before the work is to be performed. B) the painter, because since she first raised the issue of order of performance, she has a right to establish that order. C) the owner, because where a contract is silent about the order of performance, it is too indefinite to be enforced. D) the owner, because where an agreement is silent about the order of performance, payment is not required until after the work is completed.

D) the owner, because where an agreement is silent about the order of performance, payment is not required until after the work is completed.

Owner and Builder entered into the following written agreement: "Builder agrees to construct a three bedroom, single family residence on Owner's property [described], in exchange for $75,000, payable on completion of the structure. Owner will pay Builder an additional $5,000 on October 15, provided that the residence is completed before October 1." Builder commenced construction of the residence and completed it on September 25. Owner paid Builder $75,000 on September 26, but stated at the same time, "I'm never going to pay you another $5,000 for this house." The contract provision which states that Builder will construct the residence by October 1 can be most accurately characterized as: a) An express condition precedent to Owner's obligation to pay an additional $5,000. b) A concurrent condition to Owner's obligation to pay an additional $5,000. c) A constructive condition precedent to Owner's obligation to pay an additional $5,000. d) An obligation, breach of which will entitle Owner to damages.

a) An express condition precedent to Owner's obligation to pay an additional $5,000.

A sailor was out in his small sailboat in the channel between the shore and an offshore island when he negligently caused his boat to become swamped and to sink. The sailor was thrown into the ocean, but was held afloat by his flotation jacket. About thirty minutes later, a yachtsman came along in his yacht, on his way to a party on the island. He saw the sailor in the water and stayed long enough to determine that the sailor was not injured. "Someone else is bound to come along and I'm late for my party. I'll advise the Coast Guard when I reach the island" said the yachtsman as he sailed away without picking up the sailor. About 30 minutes later, the yachtsman reached the island and notified the authorities of the sailor's position. When the Coast Guard reached the sailor, they discovered that he had been run over and severely injured by a powerful ocean racing boat operated by a captain. The captain had seen the sailor in time to avoid hitting him, but was sufficiently intoxicated so that he was unable to steer the boat aside quickly enough. The sailor brought an appropriate action against the yachtsman to recover for the injuries he suffered in the water. What result? a) Judgment for the yachtsman, because he did not leave the sailor in greater peril than he found him. b) Judgment for the yachtsman, because the sailor was responsible for his own predicament due to his own negligence. c) Judgment for the sailor, because the yachtsman incurred a duty to rescue the sailor by stopping and investigating the sailor's situation. d) Judgment for the sailor, if a reasonable person would have picked up the sailor under the same circumstances.

a) Judgment for the yachtsman, because he did not leave the sailor in greater peril than he found him.

A researcher was a tomato genetics stock researcher who had developed and produced a variety of square tomatoes. The researcher believed that this would revolutionize sandwich-making. On May 1, the researcher orally agreed to sell five tons of this square tomato to a broker for $5,000. The $5,000 was payable on May 31. The broker orally agreed to pay $2,000 of the purchase price to a landlord for back rent owed by the researcher. On May 5, the researcher dictated the agreement to his secretary. When the secretary typed up the contract, she failed to include any provisions relating to the payment to Landlord. In an action by the landlord against the broker for $2,000, which of the following, if proved, would be the broker's strongest defense? a) On May 3, before the landlord was aware of the oral agreement between the researcher and the broker, the researcher agreed with the broker not to pay any part of the purchase price to the landlord. b) Whatever action the landlord may have had against the researcher was barred by the statute of limitations prior to March 1. c) Before he instituted his action against the broker, the landlord had not notified either the broker or the researcher that he had accepted the subject arrangement for paying the landlord. d) There was no consideration to support the researcher's antecedent promise to pay the landlord the $2,000.

a) On May 3, before the landlord was aware of the oral agreement between the researcher and the broker, the researcher agreed with the broker not to pay any part of the purchase price to the landlord.

Oscar owned a vacant lot upon which he wished to operate a gas station. He entered into a written contract with Felix on January 15 in which Felix agreed to construct a gas station according to specifications incorporated into the writing, in exchange for $250,000. The agreement provided that a progress payment of $100,000 would be made when construction was half complete, and the remainder of the contract price would be paid upon completion. The terms of the writing also specified that work was to begin on March 15 and was to be completed no later than September 15 of the same year, "because Oscar has irrevocably committed to begin purchases of petroleum products on October 1." On March 1, Felix telephoned Oscar and stated that due to unanticipated delays in another project, he could not begin construction of the gas station until June 1, and could complete construction no earlier than December 1. When informed that the two-and-a-half month completion delay would result in unacceptable money losses to Oscar, Felix said, "You've got to give me until December 1 or I can't do the job."Without further communication with Felix, on March 10 Oscar entered into a contract with Carl to construct a gas station on Oscar's land under the same terms as the Oscar-Felix contract, except that work was to begin April 1, and the total contract price was $300,000. On March 20, Felix appeared at Oscar's lot and offered to begin construction of the gas station with completion to be by September 15, but Oscar refused to allow him to proceed. On April 1, Carl commenced construction and subsequently completed the gas station on September 10. If Oscar brings an action for breach of contract against Felix and it is determined that Felix is liable to Oscar for breach of their agreement, what damages should Oscar recover in a breach of contract action against Felix? a) The cost of completion minus the contract price; or $50,000. b) The market value of the gas station as completed minus the contract price. c) The market value of the gas station as completed minus the contract price, plus any incidental damages resulting from the delay in completion. d) The cost of completion, or $300,000.

a) The cost of completion minus the contract price; or $50,000.

A textile merchant acquired 1000 bolts of stretch velvet. The merchant knew that a well-known women's designer planned to center his spring collection around stretch velvet. The merchant also knew that an unexpected textile strike resulted in a shortage of stretch velvet. The merchant sent the designer a fax saying: "I am sending you 1,000 bolts of stretch velvet at $50 per bolt." The merchant shipped the first 300 bolts to the designer. Upon receipt of the merchant's fax, the designer phoned the merchant and said "I accept your offer." 5 days later, after using the first 300 bolts, the designer orally repudiated the contract. If the merchant sues the designer for breach of contract by refusing to take 1,000 bolts of stretch velvet and the designer defends by pleading the Statute of Frauds, which of the following is the most accurate statement of the law? a) The designer is liable for 300 bolts, but the defense is good as to the remaining 700 bolts. b) The defense fails because, as between merchants, the merchant's telegram is a memorandum sufficient to meet the requirements of the Statute of Frauds. c) The defense fails because the Statute of Frauds does not apply to this transaction. d) The defense is good as to all 1,000 bolts.

a) The designer is liable for 300 bolts, but the defense is good as to the remaining 700 bolts.

An event planner who lived in State X wanted to have a lavish event at a fancy hotel located in State Y. She rented out half the hotel for the entire weekend of the event. The contract for these arrangements specified that any action between the parties must be brought in the state court of State Y. One month before the event the hotel decided to undergo a major structural retrofit and terminated the contract with the planner. The planner brought suit against the hotel in the state court of State Y. The hotel immediately filed for removal to federal court, and the removal was granted. The planner then filed a motion for remand to the federal judge assigned to hear the action, in order to remand the case back to state court, per the parties' contract. Which of the following statements is true? a) The planner must file the motion for remand within 30 days of the notice of removal b) Planner has the burden of proof to succeed in a motion for remand. c) The motion for remand must be brought before the state judge, not the federal judge. d) Courts generally will not uphold contractual clauses that are contrary to removal orders.

a) The planner must file the motion for remand within 30 days of the notice of removal

On April 1st, Nurseryman offered to sell 500 ready-to-plant navel orange trees to Grower for $2,500. Nurseryman gave Grower a signed written statement which recited the offer and stated that Nurseryman promised not to revoke for a period of 45 days. Two weeks later, on April 14th, Grower wrote to Nurseryman: "I have decided to purchase the trees, and I have enclosed my check for $2,500. I am not going to be able to plant the trees until September, and so I will accept delivery at that time. I will, of course, pay you for their water and maintenance." Grower's letter is: a) An acceptance, and Nurseryman must retain and care for the trees but is entitled to the reasonable value of that service. b) An acceptance, and Nurseryman may refuse to retain and maintain the trees. c) A counter-offer, because it changes the terms of the offer. d) A counter-offer, because it was not a definite expression of acceptance.

b) An acceptance, and Nurseryman may refuse to retain and maintain the trees.

In an effort to boost declining customer traffic, Harmony Mall installed a carousel. Being the only carousel for hundreds of miles, the Mall owners hoped children would drag their parents to the mall. The Mall hired Ultratechnics to install, operate, and maintain the carousel. The plan worked beyond anyone's expectations as traffic in the mall increased 200%. One day, five-year-old Briana came to the mall with her mother. After her mother had finished shopping, Briana got her promised ride on the carousel. As Briana was boarding the carousel, she tripped and fell against some of the mechanism which made the horses go up and down. Someone had neglected to replace the protective housing that normally covered the machinery. Briana's hand was caught in the moving gears. A Harmony Mall employee, Theresa, was walking by the carousel when Briana fell and was the closest person to the scene. Had she grabbed Briana quickly, Briana would only have suffered superficial cuts and bruises. Instead, Theresa stared motionlessly at Briana, mouth gaping in amazement. As a result of the delay, Briana's hand was pulled further into the gear mechanism and three fingers were broken. If Briana files suit against Harmony Mall through an appropriate representative, who will prevail? a) Briana, but only if Theresa failed to act as a reasonable person. b) Briana, if an employee of Ultratechnics was negligent in not replacing the engine housing. c) Harmony Mall, because Ultratechnics' acts were the cause of Briana's injury. d) Harmony Mall, because Briana's mother assumed the risk of letting her ride the carousel.

b) Briana, if an employee of Ultratechnics was negligent in not replacing the engine housing.

Tom purchased an elaborate remote control model airplane for his son, Jason, for Jason's ninth birthday. Although called "model," this plane had a three-foot wingspan and was capable of every maneuver a real stunt plane could negotiate. Tom instructed Jason at length regarding use of the plane and only permitted him to use it under Tom's direct supervision at a nearby field set aside for the use of model plane enthusiasts. One afternoon, school was let out unusually early. Jason headed home but was terribly bored and, disregarding his father's instructions, removed the plane from its cabinet in the garage and began playing with it in his front yard. With the plane soaring in circles above his house, Jason became confused by the controls. He sent the plane peeling down the street at about a five foot height. Lisa, a pedestrian on the adjoining sidewalk, panicked when she saw the plane headed directly towards her. Scrambling into Jillian's yard in an attempt to escape the plane, she fell over a large ceramic sleeping deer, breaking her hip. If Lisa brings suit against Jason, will she recover? a) Yes, but only if Jason was engaged in an adult activity. b) Yes, if Jason did not behave as a reasonably prudent child of the same age, knowledge, and experience under the circumstances. c) No, because a nine-year-old child is presumed incapable of the state of mind necessary for negligence. d) No, if she has already obtained a judgment against Jillian.

b) Yes, if Jason did not behave as a reasonably prudent child of the same age, knowledge, and experience under the circumstances.

Flush from their victory in the company bowling league and from half a dozen beers they each consumed in the bowling alley bar, Teddy and Albert tore across town around 1:00 a.m. in Teddy's Grand Am automobile with Teddy at the wheel. For sport, Teddy and Albert proceeded down Slauson Avenue, an almost exclusively commercial district, playing a game where at each successive cross street Teddy tried to push the car five miles per hour faster. By the time they passed Maple, Teddy was going 70 mph in a 40 mph zone.Utterly preoccupied with a personnel problem in his business, Jacob stepped into the street to cross Slauson without looking. Halfway across Slauson, Jacob heard the screeching of brakes, looked up to see Teddy's car headed towards him, and tried to flee. Teddy hit Jacob's left leg when he lost control of the Grand Am in a skid. Had Teddy been going less than 50 mph, he would have been able to stop in time to avoid hitting Jacob.Jacob brings suit against Teddy. The jurisdiction in which this accident occurred has not adopted any form of comparative negligence. Will Jacob recover damages against Teddy? a) Yes, if Teddy was negligent in exceeding the speed limit in a commercial district late at night. b) Yes, if Teddy was reckless. c) No, if Teddy did not have the last clear chance to avoid the injury to Jacob. d) No, because Teddy did not know to a substantial certainty he would hit Jacob.

b) Yes, if Teddy was reckless.

Ever since Zack was eight years old, he has been an avid model airplane builder and collector. He is now seventeen and has been buying and selling model airplanes for the past five years. Ben, a forty-five year old security guard who had never been interested in model airplanes before, saw Zack flying a radio controlled model airplane one Sunday in a park and decided that he would like to own one. Ben asked Zack if he knew where he could get such a model, and Zack showed Ben an airplane he had in the back of his Land Rover. Ben arranged to come by Zack's house the following Saturday to bring Zack $150 cash and to pick up the model airplane. Zack wrote the terms of their agreement on a scrap of paper he found in his front seat and both parties signed it. However, when Zack wrote the terms of the agreement on the scrap of paper, he had made a mistake in describing the model's serial number. Instead of writing BX-301, the serial number of the model he had agreed to sell to Ben, Zack had written BX-305, the serial number of another model of the same airplane that he had at home. Neither Zack nor Ben had noticed the mistake until Zack tendered to Ben the airplane with the serial number BX-301. Ben refused to take it. Zack then tendered to Ben the airplane with the serial number BX-305, but Ben refused to take that one as well. Zack sued Ben for breach of contract for his refusal to accept the airplane with the serial number BX-305. Which party will win? a) Zack, provided the two airplanes were of equal or near equal value. b) Zack, because the parol evidence rule bars extrinsic evidence that the airplane identified in the writing is not the one Ben agreed to accept. c) Ben, because parol evidence may be admitted to show that Ben never agreed to buy the model airplane with the serial number BX-305. d) Ben, because the writing was insufficient to satisfy the Statute of Frauds.

c) Ben, because parol evidence may be admitted to show that Ben never agreed to buy the model airplane with the serial number BX-305.

A plaintiff brought an action in federal court for battery against a diverse defendant claiming $10,000 in actual damages, $12,000 in pain and suffering, $15,000 in lost wages, and $39,000 in punitive damages, for a total of $76,000. The defendant filed a motion to dismiss. How should the court rule on the motion? a) Grant the motion, because punitive damages cannot be used towards the amount in controversy for a case arising out of diversity. b) Grant the motion, because although punitive damages are counted towards the amount in controversy, they cannot account for the majority of the total amount. c) Deny the motion, because the amount in controversy is satisfied. d) Deny the motion, because a plaintiff can aggregate multiple claims against a single defendant.

c) Deny the motion, because the amount in controversy is satisfied.

Chemco is engaged in the business of manufacturing hydrogen cyanide, a potentially deadly gas which is used by industries utilizing metallic plating processes. Attached to every canister of hydrogen cyanide sold, Chemco encloses a detailed set of safety warnings describing proper storage techniques and warning of the severe health hazards posed by improper handling. Plateco, a metallic plating business, purchased some hydrogen cyanide canisters from Chemco. When Plateco's employees attempted to transfer the gas to Plateco's storage facilities, some of the gas escaped. The gas fumes drifted out of Plateco's factory and onto the adjacent sidewalk. Walker was strolling by on the public sidewalk, inhaled the gas and died from breathing the gas. If Walker's heirs bring a wrongful death action against Chemco based on strict liability for defective product, how should the court rule? a) For Walker, because hydrogen cyanide is an inherently dangerous product. b) For Walker, because manufacturers of toxic chemicals are strictly liable. c) For Chemco, because the product was not defective. d) For Chemco, because the product was not under Chemco's control.

c) For Chemco, because the product was not defective.

The Metropolitan Transportation Corporation (Metro) was licensed by the City of Metropolis to provide mass transit services in the city. Metro operated a fleet of electric trolleys throughout Metropolis; power was provided by a "third rail" which carried high voltage electrical current. The power rail was heavily insulated—the electrical current was transferred to the trolleys by an induction process which did not require actual contact between the trolley and the power line. Because the material which insulated the "third rail" contained a small amount of platinum necessary to facilitate the induction process, thieves often removed the insulating material and took it away in order to extract the platinum for resale. Despite the fact that Metro used sophisticated electronic sensors to detect when insulation was missing, and also conducted frequent patrols by special security guards specifically assigned to search for insulation breaks, insulation thefts were frequent and occurred everywhere there were Metro tracks. In one part of Metropolis, the Metro tracks passed between a residential neighborhood and a city park. Although a single elevated walkway provided access between the two areas, the children of the neighborhood often took shortcuts by climbing the low fence separating the tracks from the surroundings and crossing the tracks to and from the park. One Saturday, when thieves had stolen "third rail" insulation from this area, six-year-old Samuel was crossing the Metro tracks on his way to play in the park when he stepped on the exposed "third rail" and was severely burned by electrocution. Samuel's legal representative brought an appropriate action on his behalf against Metro to recover for the injuries he had suffered from the exposed power rail. What result? a) Judgment for Metro, if it used reasonable care in guarding against, discovering and repairing stolen "third rail" insulation. b) Judgment for Metro, because the intentional criminal act of stealing "third rail" insulation was an intervening, superseding cause of Samuel's injuries. c) Judgment for Samuel, if Metro knew that children crossed the tracks and could readily have prevented this. d) Judgment for Samuel, because Metro is strictly liable for any injury resulting from its use of high voltage electricity.

c) Judgment for Samuel, if Metro knew that children crossed the tracks and could readily have prevented this.

Rusty made his living by stealing other people's cattle and selling the meat to unscrupulous butchers. He drove his truck and trailer to Vasco's ranch one evening because he had heard that Vasco was famous for his cattle. Rusty set up his trailer and climbed into a field intending to steal a cow. Unknown to Rusty, the field contained Vasco's prize fighting bull, used in Portuguese-style bullfighting. The bull charged Rusty and knocked him to the ground, breaking his ankle in the process. Tyrone, who happened to be driving by Vasco's property, saw Rusty fall and, knowing of Vasco's bull, realized that Rusty was in extreme danger. Tyrone stopped his truck, got out and vaulted the fence into the field. Waving a red bandanna to distract the bull, Tyrone shouted to Rusty, "I'll draw him off. You get out of the field!" The bull charged Tyrone, who spent an exciting few minutes dodging the enraged animal while Rusty hobbled to the fence and climbed over. As Tyrone abandoned his bandanna and leaped over the fence, his foot caught on the boards and he fell out of the field, breaking his leg. Tyrone brought an appropriate action against Vasco for personal injuries suffered in the incident. What result? a) Judgment for Vasco, because he was not negligent under the circumstances. b) Judgment for Vasco, because Tyrone knew that the bull was dangerous and voluntarily exposed himself to the danger. c) Judgment for Tyrone, because Vasco is strictly liable for injuries caused by his animal. d) Judgment for Tyrone, because he was not negligent in effectuating the rescue of Rusty.

c) Judgment for Tyrone, because Vasco is strictly liable for injuries caused by his animal.

Two plaintiffs, citizens of Georgia, brought an action in the United States District Court in New Jersey against Defendant 1, a New Mexico corporation, and Defendant 2, a New Jersey corporation. Defendant 1's only office and manufacturing plant are located in New Mexico. At no time has Defendant 1 had an office or salesperson in New Jersey. Defendant 2's sole place of business is in New Jersey. Plaintiffs allege that they were seriously injured when a display wardrobe model tipped over in Defendant 2's store. Each plaintiff requested damages in the sum of $80,000. The wardrobe had been manufactured by Defendant 1 and shipped to a third party in Georgia who had a contract with Defendant 1 to act as exclusive distributor of its furnishings in six states, including New Jersey and Georgia. Process was served personally on the president of Defendant 1 at its office in New Mexico and on the president of Defendant 2 at its office in New Jersey. Thereafter, Defendant 1 filed a cross-claim against Defendant 2 for $120,000 alleged to be due for merchandise previously sold by Defendant 1 to Defendant 1.How should the court rule on a motion to strike Defendant 1's cross-claim? a) The court should deny the motion, because Defendant 1's action might confuse or divert the jury with respect to the original claim. b) The court should deny the motion, because there is diversity subject-matter jurisdiction with respect to Defendant 1's cross-claim against Defendant 2. c) The court should grant the motion, because the cross-claim is unrelated to the plaintiffs' action. d) The court should grant the motion, because there is no supplemental jurisdiction with respect to this action.

c) The court should grant the motion, because the cross-claim is unrelated to the plaintiffs' action.

Potter and Daphne entered into a written contract whereby Potter agreed to manufacture for Daphne, and Daphne agreed to buy 100 crates of matches at a price of $75 per crate. The contract specified that, "it is expressly agreed that Daphne shall be under no obligation under this contract unless 100 crates of matches are delivered to her at her place of business no later than November 1." Eighty crates of matches, meeting Daphne's specifications, were tendered by Potter on November 1. The remaining twenty crates of matches were tendered on November 5. Daphne refused to accept any of the crates of matches. If Potter sues Daphne, which of the following contentions would best support Potter's case, assuming each contention is factually sustainable? a) Delivery of the 100 crates of matches November 1 was delayed by a truckers' strike, and was not Potter's fault. b) A drop in Daphne's credit rating from "good" to "fair" had caused Potter not to produce and tender the full 100 crates of matches before November 1. c) The specified matches were a type of matches produced only by Potter and manufactured by Potter especially to meet Daphne's needs and specifications. d) Daphne had orally agreed, just prior to the time the written contract was executed, to accept and pay for partial deliveries of the matches.

c) The specified matches were a type of matches produced only by Potter and manufactured by Potter especially to meet Daphne's needs and specifications.

In order to demonstrate the inadequacy of most witnesses' perceptions to her trial practice class, a professor arranged a phony attack scene with her husband. The husband was to enter the classroom, yell some obscenity at the professor, and then charge at her, seeming to be attempting to choke the professor. The professor would then appear to subdue her husband with a martial arts hold. The scene was to end with the husband fleeing and the professor directing the class to write one paragraph describing the details of the incident. All went as planned during the scene until the professor was "subduing" her husband with the martial arts hold. At that moment, a security officer entered the classroom, and believing that the professor had finally gone berserk, yelled a warning and drew a gun he legally carried. To evade the shot, the professor dove to the floor behind a table, knocking a student from her chair in such a fashion that the student was hit by the security officer's single shot. According to the common law, if the student sues the security officer for battery, will she prevail? a) No, because the security officer did not know to a substantial certainty that a touching of the student would occur. b) No, because the security officer reacted reasonably to the danger apparently posed to the husband. c) Yes, because the security officer intended to place the professor in apprehension of a touching. d) Yes, because a reasonable security officer would not have fired his revolver in a classroom setting.

c) Yes, because the security officer intended to place the professor in apprehension of a touching.

Three parties formed a partnership, but their relationship soured and they became involved in litigation. One of the parties filed suit in federal court in State A. One of the defendants challenged the federal court's diversity jurisdiction, arguing that complete diversity did not exist. For the purpose of determining diversity jurisdiction, which of the following parties is not domiciled in State A? a) A party lives in State A, but often stays in temporary housing in State B which adjoins State A, and travels every day to State A for work. b) A party stays with close friends in State B for more than four months while considering whether to buy a house there and maintains their home in State A while doing so. c) A party rents and lives in an apartment in State A but travels to adjoining State B every day to work. d) A party owns and rents out a townhouse in State A but lives in adjoining State B.

d) A party owns and rents out a townhouse in State A but lives in adjoining State B.

Sandra runs a small corner hardware store. Sandra sells standard framing nails in 5 pound bags. The bags are not marked as to the weight of nails they contain, but all of her customers know that Sandra sells nails by the 5 pound bag. Sandra learns that the new clerk at the store has mistakenly been filling the bags with only 4 pounds of nails. Rather than repackage the nails, Sandra puts all of the bags with only 4 pounds in them on the "as is" rack in the store. The sign above the rack conspicuously reads: "Everything on this rack is sold AS IS." Gregg, a regular customer of Sandra's, buys three of these bags of nails off of the "as is" rack. He soon recognizes that the bags have only 4 pounds of nails in them. Gregg sues Sandra for fraud. Which if the following is most accurate? a) Gregg should prevail because Sandra should have revealed that the bags contained only 4 pounds of nails. b) Gregg should prevail despite the fact that he bought the nails on an "as is" basis. c) Gregg should not prevail because he should not have relied on the fact that the bags ordinarily have 5 pounds in them when deciding to make this purchase. d) Gregg should not prevail despite the fact that Sandra concealed the fact the bags had only 4 pounds of nails.

d) Gregg should not prevail despite the fact that Sandra concealed the fact the bags had only 4 pounds of nails.

An owner of a sports car returned home from work and mistakenly left the keys to the car in the vehicle. A thief passing by looked into the sports car, saw the keys, and drove off with the sports car. Two blocks away, the thief ran through a red light and collided with a sedan driven by a driver. Mistakenly believing that he was responsible because he owned the sports car, the owner told the driver: "I promise to be responsible for all damages which the thief caused you." If the driver brings an action to recover for costs expended in medical treatment for a broken arm sustained during the accident, the owner's best defense is: a) Mistake of fact as to basic assumption. b) Statute of Frauds. c) Indefiniteness of the owner's promise. d) Lack of consideration.

d) Lack of consideration.

The police chief of a small town told some police officers and a magistrate to put together a bogus warrant to search a man and woman's apartment while the woman was out of town. The police came to the man's door with the warrant, and conducted the search, yet found nothing. The man saw the warrant contained untrue information, and filed a Section 1983 claim against the chief in federal district court based on the chief's alleged violation of his constitutional rights. He sent his hulking,19-year-old son to serve process on the chief. The chief received the documents but immediately threw them away, believing that they were for a football team fundraiser. The man then conducted an interview with the local magazine regarding his suit, and the magazine called the chief for comment. The chief replied that they had not heard of the suit until now and had not been served. The journalist informed the chief of the details of the suit. If the chief later files an answer to the man's complaint arguing that service was defective and thus the case cannot go forward, will he prevail? a) Yes, because the man failed to properly serve process on the chief. b) Yes, because the chief did not read the documents that were served on them before throwing them out. c) No, because the chief failed to make their objection regarding service in a motion to dismiss. d) No, because the chief was properly served.

d) No, because the chief was properly served.

A businessman owns a luxury hotel which he is in the process of remodeling. The businessman entered into a contract with a tile contractor under which the tile contractor agreed to install new tile in all 200 of the hotel's rooms. Since the businessman desired only the best for his hotel, the tile specified under the contract was very expensive. In addition, the tile had to be specially ordered because the contract required each tile to be embossed with a unique pattern designed specifically for the hotel. The contract specified that upon completion of the job, the businessman was to pay the tile contractor $800,000. The tile contractor ordered and received delivery of the specified tile at a cost of $300,000. However, before the tile contractor began installing the tile, the businessman informed the tile contractor that he had decided "to go in a different direction," and repudiated the contract. The tile contractor sued the businessman for damages. At trial, neither the tile contractor nor the businessman was able to establish the cost of completing the project because it was too speculative under the circumstances. Disregarding incidental or consequential damages, how much may the tile contractor recover in an action against the businessman? a) Expectation damages in the amount of $800,000 (the contract price), since the tile was special ordered and cannot be used for another project. b) Expectation damages in the amount of $500,000 (the contract price, less the cost of the tile). c) Restitution damages in the amount of $300,000. d) Reliance damages in the amount of $300,000.

d) Reliance damages in the amount of $300,000.

A large technology company entered the online music business, and offered free digital downloads of certain musicians' work in order to attract music fans to its Web site. The company reimbursed the artists for the downloads but did not enter into agreements with the artists allowing for their work to be downloaded for free. A singer, upset that their music would be devalued as a "free giveaway," filed suit in federal district court demanding compensation for the reputational damage suffered by having their work be given away for free. As the case is getting ready to go to trial, a bass player whose music was also being given away for free by the website attempted to intervene to join the action, and provided evidence that his action against the company was based on the same questions of law and fact as the singer's action. Which of the following best describes how the court will rule on the attempt to intervene? a) The court must allow the bass player to intervene in the action. b) The court must not allow the bass player to intervene in the action. c) The court will likely allow the bass player to intervene in the action, because the bass player is asserting a protectable interest relating to the property or transaction involved in the lawsuit and shares a common question of law or fact with the singer's. d) The court will likely not allow the bass player to intervene in the action, because doing so could unduly delay and prejudice the existing parties claims.

d) The court will likely not allow the bass player to intervene in the action, because doing so could unduly delay and prejudice the existing parties claims.

A woman was standing at the perfume counter near the west door of a department store when a man dashed through the door, shoved her to the ground, and wrenched her purse from her arm. The perfume counter clerk took up pursuit, only to see the man jump into a waiting car and flee.At closing that day, the clerk's cash drawer was $1,000 short. The clerk knew that the woman's son had previously been convicted of petty theft, and although she had not seen the son with the woman, immediately concluded that the man and the woman had conspired to create a diversion while the son pilfered the cash drawer. The clerk called the department store security and stated, "the woman is a thief. She took $1,000 from my cash drawer during that incident today." The clerk said this even though a reporter for a newspaper was standing alongside her at the time. The reporter had just completed an interview with the clerk about shoplifting problems. The clerk's supervisor subsequently confessed to the theft of the $1,000. If the woman sues the department store for defamation, what will be the probable outcome? a) The department store will win because thefts are a matter of public concern. b) The department store will win because the clerk was privileged to make the statement to the security office. c) The woman will win because there was a publication. d) The woman will win because the clerk knew the newspaper reporter was standing alongside her.

d) The woman will win because the clerk knew the newspaper reporter was standing alongside her.


Set pelajaran terkait

Chapter 19 Politics and Economics

View Set

NUR 1068 Health Assessment- Ch 1 - collecting and analyzing data

View Set

bio 111 final (all unit quizzes)When a car burns gasoline, much of the energy is released in the form of heat. Which of the following best describes this process in relation to the first law of thermodynamics?

View Set